Sunteți pe pagina 1din 64

- ISTORIA MATEMATICII -

„ Numerele perfecte ca și oamenii


perfecți, sunt foarte rare”.
Descartes
(1596- 1650)

1. Istoria matematicii
RENE DESCARTES ( 1596 – 1650)
de Adrian Stan, Buzău

Matematicianul şi filozoful francez Rene Descartes (31.03.1596 –


11.02.1650) publică în 1637 lucrarea sa „ Discurs despre metoda de a conduce
raţiunea şi de a căuta adevărul în ştiinţe...”, punând astfel bazele filozofiei
moderne bazate pe cunoaştere şi pe certitudini matematice așa numitul sistem de
filozofie cartezian, (de la numele său latin, Cartezius), ce a influențat ultima
jumătate de secol al XVII-lea. Sistemul său filozofic era unul inovator bazat pe
observație și experiment.
Cuvintele rămase celebre de la el, ” Dubito ergo cogito. Cogito ergo
sum” (”Mă îndoiesc, deci cuget. Cuget deci exist ”) reprezintă chiar esenţa
sistemului său filozofic, şi anume, în demersul său de a găsi adevărul filozofic el
supune percepţiile senzoriale şi afirmaţiile despre om şi despre lume unui proces
elementar de scepticism cum ar fi că „ simţurile pot înşela, lumea exterioară poate fi stimulată, adevărat este
însă numai ceea ce poate fi clar recunoscut”.
Contribuțiile sale matematice sunt imense și pe drept cuvânt este considerat părintele geometriei
analitice și reformatorul algebrei. El a dat metoda de rezolvare a problemelor de geometrie cu ajutorul
metodelor algebrice și invers, raportând măsurile figurilor la un sistem de referință XOY cu ajutorul
coordonatelor.
Încă de la vârsta de 8 ani era poreclit „filozoful” căci dădea dovadă de o bună stăpânire a cunoștințelor
iar de la 14 ani era preocupat de studii matematice. Însă, întâlnirea de la Paris cu viitorul său prieten
Mersenne ( 1588-1648), l-a determinat să aprofundeze mult mai mult matematica. Își ia licența în drept la
Universitatea din Poitiers în 1617 după care se înrolează în armata prințului de Orania. În acest timp, în
noiembrie 1618 se întâlnește cu matematicianul Isaac Beeckman( 1588-1637) care-i încurajează spiritul
pentru studiul matematicii și-i stimulează gustul pentru invenții. De aceea, lucrarea sa din 1618,
“Compendium Musicae” îi este dedicată prietenului său Beeckman.
Descartes scrie în 1630 lucrarea “Les Metheores”, unde prezintă o serie de fenomene legate de
meteoriții observați de el la Roma, aducând în prim plan o așa numită “teorie a curcubeului”.
Prin cartea “La Dioptrique” (1631) tratează teoria fizico-matematică a instrumentelor optice și tratează
și legea refracției, descoperită independent de Willebrrord Snellius (1581 -1626). El descrie și funcționarea
ochiului considerat ca o lentilă.
Cartea “La Geometrie”(1637) ( începută în 1619) este fundamentală pentru geniul său matematic, căci
aici, este primul matematician care introduce noțiunea de variabilă și folosește metoda coordonatelor prin care
problemele de geometrie se reduc la probleme de algebră. A folosit literele alfabetului latin pentru notații ,
constantele cu primele litere iar variabilele cu ultimele litere. Astfel, geometria analitică pe care el a fondat-o
este în strânsă legătură cu sistemul de coordonate carteziane - de la numele lui, pe care el l-a introdus și unde
a notat axele cu Ox și Oy. Astfel, un punct din plan se reprezintă prin două coordonate, abcisa pe Ox și
ordonata pe Oy. Luând un segment al dreptei ca fiind segment unitate, a descoperit că pentru orice punct de pe
- ISTORIA MATEMATICII -

dreaptă se poate asocia un număr real, introducând astfel axa numerelor reale
ceea ce era o noutate pentru acel timp.
De asemenea, a folosit exponenții la calculul cu puteri așa cum îi folosim și noi astăzi, a dat o metodă de
rezolvare a ecuațiilor generale de gradul al patrulea, a descoperit relația dintre numărul fețelor, vârfurilor și
muchiilor unui poliedru convex și a studiat problema inversă a tangentelor – punctul de plecare în studiul
calculului diferențial realizat de Leibniz și Newton.
Întodeauna, Descartes nu a vrut să ia parte la diverse controverse sau să fie implicat în scandaluri și de
aceea , de exemplu, când a vrut să publice cartea sa “Lumea”, (“Le Monde”) în 1663, a aflat de condamnarea
lui Galileo Galilei de către Inchiziție datorită promovării teoriei copernicane a heliocentrismului pe care
însuși Descartes o promova în tratatul său, așa că a fost nevoie să amâne publicarea cărții sale până în 1664.
În 1644 apare cartea “Principia Philosophiae” (Principiile filozofiei) unde are preocupări din domeniul
filozofiei și cosmogoniei, de altfel , de la el rămânând foarte multe scrisori de
filozofie purtate între el și Mersenne , cu principesa palatină Elisabeth de
Phalz (1618 - 1680) sau cu ceilalați oameni de știință, unele din scrisori fiind
descoperite chiar și cu mult timp după moartea sa.
Rene Descartes a fost inspirat de opera lui Francois Viete ( 1540-1603)
și a contribuit la stabilirea noțiunii de număr algebraic, noțiune care mai târziu
a fost dezvoltată pe baze solide de către Gauss, Abel. Aceste numere erau
însoțite de semnele + și - care ulterior a impus și introducerea noțiunii de
valoare absolută și relativă a unui număr.
El a arătat că la o ecuație se poate stabila numărul de soluții pozitive sau
negative fără să se resolve aceasta și este de asemenea, a arătat că pentru
unele ecuații există rădăcini de forma a+bi, cu a, b  , i 1 , care nu se
pot reprezenta într-un sistem de axe de coordonate, fiind un precursor al lui Gauss care le va considera numere
imaginare sau complexe și care va constitui o întreagă teorie matematică pe seama lor.
În 1649, prin intermediul ambasadorului Franței în Suedia, Descartes
ajunge la curtea reginei Christina (1626-1689) care devenise una din cei
mai importanți monarhi ai Europei, pentru a-i preda lecții de filozofie. Însă,
vremea foarte rece de acolo nu i-a fost prielnică, el având deja o stare
precară a sănătății, astfel că, criza de pneumonie i-a fost fatală, decedând pe
11 februarie 1650. Trupul său a fost adus în Franța în 1667 fiind depus în
biserica Saint-Germain –des – Pres din Paris.
În amintirea sa, un asteroid a fost denumit cu numele său.

Bibliografie:
1. N. Mihăileanu. Istoria matematicii. Vol I, II. Editura Didactică și Pedagogică. București. 1981.
2. Vasile Bobancu. Caleidoscop Matematic. Editura Niculescu. București. 2001.
3. www.wikipedia.com
Prof. Liceul Tehnologic “Costin Nenițescu”, Buzău
- ARTICOLE ȘI NOTE MATEMATICE -

„Nu pot exista lucruri, oricât de îndepărtate, la care să


nu ajungem, şi oricât de ascunse, pe care să nu le descoperim”.
Descartes
(1596- 1650)

2. Articole și note matematice


Asupra inegalităţilor de tip Ionescu-Weintzenböck
de Constantin Rusu, Râmnicu Sărat
I. Introducere.
Inegalitatea 4 3 ˜ S d a 2  b 2  c 2 , (1), a, b, c fiind laturile unui triunghi ABC şi S aria sa este
inegalitatea Ionescu-Weintzenböck (I-W) (vezi [1]).
S-au dat foarte multe demonstraţii (peste 44). Acest fapt subliniază importanţa acestei inegalităţi.
Ion Ionescu (1870-1946) a fost ctitorul Gazetei Matematice, ctitorul S.S.M.R, ctitorul învăţământului
politehnic din România, publică inegalitatea în G.M. octombrie 1897, pag. 52.
Roland Weitzenböck (1885-1955) publică aceeaşi inegalitate în 1919, în revista Mathematische-Zeitschrift.
Denumirea actuală a inegalităţii s-a produs foarte târziu şi anume, în ianuarie 2013 (vezi [1]).
II. Prezentăm acum două demonstraţii ale acestei inegalităţi, precum şi o interpretare geometrică a ei.
xy sin A x
II.1. Cu notaţiile: b x, c y, a x 2  y 2  2 xy cos A , S , x ! 0, y ! 0, t , relaţia
2 y
(1) devine x 2  y 2  xy cos A t 3xy sin A œ t 2  t (cos A  3 sin A)  1 t 0 , (2).
Relaţia (2) este adevărată deoarece
' 1  2 sin 2 A  2 3 sin A cos A  4 1  cos 2 A  3 sin 2 A  3
§ S· § S·
2  2 cos¨ 2 A  ¸ 4 cos 2 ¨ A  ¸  0 .
© 3¹ © 6¹
Observăm că (2) este echivalentă cu
2
§ cos A  3 sin A · S
¨t  ¸  cos 2 §¨ A  ·¸ t 0 ,
¨ 2 ¸ © 6¹
© ¹
§ S· cos A  3 sin A
deci în (2) avem egalitate dacă şi numai dacă cos¨ A  ¸ 0 şi t .
© 6¹ 2
S
Din prima condiţie avem A , iar din a doua obţinem t 1œ x y , deci triunghiul ABC este
3
echilateral.
II. 2. O altă demonstraţie pentru inegalitatea (1) se obţine pornind de la formula
a 2b 2 c 2
OL R2  3˜ , (vezi [3]), unde O este centrul cercului circumscris 'ABC şi
(a 2  b 2  c 2 ) 2
L este punctul de intersecţie al simedianelor aceluiaşi 'ABC (punctul lui Lemoine).
a 2b 2 c 2 a 2b 2 c 2 a 2b 2 c 2
Avem OL t 0 œ R 2  3 ˜ 2 t 0 œ 3 ˜ d œ (1).
(a  b 2  c 2 ) 2 (a 2  b 2  c 2 ) 2 16S 2
a 2 sin B sin C
II. 3. În continuare, cu ajutorul formulei S , relaţia (1) se scrie echivalent astfel
2 sin( B  C )
sin( B  C ) sin B cos C  sin C cos B
4 3S d 2S ¦ œ2 3d¦ œ 2 3 ¦ (ctgB  ctgC ) œ
sin B sin C sin B sin C
œ 3 d ctgA  ctgB  ctgC , (3).
- ARTICOLE ȘI NOTE MATEMATICE -

Dar, ctgA  ctgB  ctgC ctgZ , (4), unde Z este unghiul lui Pierre René
S S
Jean Baptiste Henri Brocard (1845-1922) (vezi [2]). Deci, ctg Z t 3 œ ctg Z t ctg œ0Z d , i.e.
6 6
S
inegalitatea Ionescu-Weintzenböck are loc dacă şi numai dacă 0  Z d . Egalitatea are loc atunci când
6
S
Z , adică dacă şi numai dacă triunghiul ABC este echilateral.
6
III. Definiţie. Inegalitatea E (a, b, c) d F (a, b, c) , unde a, b, c sunt laturile unui triunghi, este de tip
(I-W) dacă 4 3S d E (a, b, c) d F (a, b, c) d a 2  b 2  c 2 .
Unei astfel de inegalităţi îi putem asocia o funcţie f : R3 o R* , f (a, b, c) E (a, b, c)  F (a, b, c) .
Demonstrăm acum următoarea
Propoziţie. Oricare ar fi numerele reale şi pozitive a şi b cu a d b şi oricare ar fi k  [0, f) avem
1
ad (1  2k )a 2  k 2 b 2 d b , (5), (vezi [4] şi [5]).
1 k
Demonstraţie. Considerăm 'AOB cu
AM
m(‘AOB) 90 0 , OA a, OB b, M  [ AB ), k , k  [0, f) .
BM
OA  k OB
Asociem punctelor A, M , B vectorii OA, OB, OM . Cu acestea OM .
1 k
2 2 2
Avem echivalenţele succesive OA d OM d OB œ OA d OM d OB œ
2 2 2
OA  2k ˜ OA ˜ OB  OB k 2
2 2
œ OA d d OB œ
(1  k ) 2
a 2  2kab  cos(‘AOB)  b 2 k 2 1
œ a2 d d b2 œ a d (1  2k )a 2  k 2 b 2 d b , q.e.d.
(1  k ) 2
1  k
Demonstrăm cu ajutorul acestei propoziţii următoarea:
Teoremă. Orice inegalitate de tip (I-W) este omogenă şi de grad 2.
Demonstraţie. Fie acum funcţia
1
g : [0, f) o [a, b) , g (k ) (1  2k )a 2  k 2 b 2 , lim g (k ) g (0) a ,
1 k k o0

k (b  a )
2 2
lim g (k ) b , g c(k ) ! 0.
k of
(1  k ) 2 (1  2k )a 2  k 2 b 2
Deducem că funcţia este bijectivă, ceea ce înseamnă că pentru orice y  [a, b) există un singur k  [0, f) ,
astfel încât y f (k ) .
Luând în inegalităţile (5) în locul lui a pe 4 3S şi în locul lui b pe a 2  b 2  c 2 , obţinem:
1
4 3S d (1  2k )(4 3S ) 2  k 2 (a 2  b 2  c 2 ) 2 d a 2  b 2  c 2
1 k
şi ţinând seama că g este strict crescătoare, avem pentru orice k1 , k 2  [0, f) , k1  k 2
1
4 3S d (1  2k1 )48S 2  k12 (a 2  b 2  c 2 ) 2 d
1  k1
1
d (1  2k 2 )48S 2  k 22 (a 2  b 2  c 2 ) 2 d a 2  b 2  c 2 , (6).
1  k2
Pentru k  [0, f) inegalităţile (6) reprezintă toate inegalităţile de tip (I-W) şi cum ele sunt omogene de grad 2,
teorema este demonstrată.
- ARTICOLE ȘI NOTE MATEMATICE -

Studiul inegalităţilor de tip (I-W) se face cu ajutorul teoremei precedente în care utilizăm notaţiile folosite în
prima demonstraţie din II.
Aplicaţie. Să se arate că inegalitatea ab  bc  ca d 4 3S  (a  b) 2  (b  c) 2  (c  a) 2 , (7) este
de tip (I-W) şi să se deducă triunghiurile pentru care ea este adevărată şi respectiv falsă.
Demonstraţie. Se ştie că 4 3S d ab  bc  ca , inegalitatea lui V.O. Gordon (1966).
Pe de altă parte 4 3S  (a  b) 2  (b  c) 2  (c  a) 2 d a 2  b 2  c 2 , este adevărată pentru orice 'ABC ,
ca fiind binecunoscuta inegalitate Hadwinger-Finsler (1937).
De asemeni este binecunoscută inegalitatea ab  bc  ca d a 2  b 2  c 2 .
În concluzie, inegalitatea (7) este de tip (I-W). Ea fiind omogenă şi de grad 2, studiul ei se face folosind
notaţiile din II şi (7), se scrie echivalent astfel
3(t  1) t 2  2t cos A  1 d 4t 2  (2 3 sin A  4 cos A  3)t  4 , căreia îi asociem funcţia
f : (0, f) o R , f (t ) 3(t  1) t 2  2t cos A  1  4t 2  (2 3 sin A  4 cos A  3)t  4 , (vezi [6]).
Pentru fiecare valoare a lui A din (0, S ) obţinem o funcţie, de unde deducem triunghiurile care verifică (7).
S
Astfel, pentru A , f (t ) 3(t  1) t 2  t  1  4t 2  2t  4 , f are un maxim egal cu 0 ce se atinge
3
pentru t 1 . Făcând graficul, deducem că mulţimea triunghiurilor care verifică (7) este
­ b ½
S ®'ABC m(‘A) 60 0 ,  (0, f)¾ .
¯ c ¿
S
Pentru A , f (t ) 3(t  1) t 2  1  4t 2  (2 3  3)t  4 , care are un maxim de coordonate
2
aproximativ egale cu t 0 # 1,0129, y0 # 0,0213 şi două rădăcini t1 # 0,8516, t 2 # 1,741.
­ 0 b ½
Din grafic deducem S ®'ABC m(‘A) 90 ,  (0, t1 ] * [t 2 , f)¾ .
¯ c ¿
Evident, inegalitatea este falsă pentru t  (t1 , t 2 ) .
Lăsăm ca exerciţiu, studiul inegalităţii (7) pentru restul valorilor A  (0, S ) .
Remarcă. Lucrarea de mai sus a fost prezentată într-o formă mult detaliată la - A XXI-a Conferinţa
Anuală a Societăţii de Ştiinţe Matematice din România, Botoşani, 11 – 14 mai 2017.

Bibliografie

1. Bătineţu-Giurgiu D.M., N. Stanciu, Inegalităţi de tip Ionescu – Weitzenböck, Gazeta Matematica, Vol.
118 , 1(2013), 1-10.
2. Mihăileanu N.N., Complemente de geometrie sintetică, Editura Didactică şi Pedagogică, Bucureşti,
1965,84-86
3. Rusu C., Calculul distanţei de la un punct al spaţiului la un punct din planul unui triunghi, Revista Escolar
de la Olimpiada Iberoamericana de Matematică, nr. 55 (Iunie-Septembrie 2016), 6-10.
4. Rusu C., Optimizări ale inegalitatăţii mediilor, Gazeta Matematică, Vol. 110, 9(2005), 444-446.
5. Rusu C., O metodă de optimizare a unei inegalităţi de numere , Articole şi note matematice, vol. I, Editura
RAFET, 2006, 32-34.
6. Rusu C., Rafinări ale inegalităţii Ionescu - Weitzenböck, a XIX-a Conferinţă anuală a S.S.M.R., Călăraşi,
30 octombrie – 1 noiembrie 2015.
7. Rusu C., Ambrinoc C., Triunghiuri ale căror laturi verifică o inegalitate de tip (I-W), a XX-a Conferinţă
anuală a S.S.M.R., Baia Mare, 19-22 mai 2016.
- ARTICOLE ȘI NOTE MATEMATICE -

În legătură cu o inegalitate algebrică din AoPS 7/2016


de Marin Chirciu, Pitești

În Art of Problems Solving 7/2016 este propusă următoarea inegalitate:


1 1
“Dacă a, b, c ! 0 și abc 1 , demonstrați că ¦ (a  1)(a  2) t 2 ”.
yz zx xy 1
Soluție: Facem substituția a
x2
,b
y2
,c
z2
și obținem M s ¦ (a  1)(a  2)
¦ x2
2
1 x4 Bergstrom

¦ § yz ·§ yz · ¦ ( x2  yz)(2 x2  yz) t
¨ 2  1¸¨ 2  2 ¸
¦ ( x2  yz)(2 x 2  yz)
©x ¹© x ¹
¦ x  2 y z
4 2 2
(1)
t
1
M d , unde
2¦ x  3¦ x yz  ¦ y z
4 2 2 2
2
(1) œ 2¦ x4  4¦ y 2 z 2 t 2¦ x 4  3xyz ¦ x  ¦ y 2 z 2 œ ¦ y 2 z 2 t xyz ¦ x , adevărată din
A2  B2  C 2 t AB  BC  CA, cu A yz, B zx, C xy . Egalitatea are loc pentru a b c 1 .
În acest articol se prezintă dezvoltări ale acestei inegalități:

1 3
1. Dacă a, b, c ! 0 și abc 1 arătați că ¦ (a  n)(a  n  1) t (n  1)(n  2) , unde 0 d n d 1 .
yz zx xy 1
Soluție: Facem substituția a
x2
,b
y2
,c
z2
și obținem M s ¦ (a  n)(a  n  1)
1 x4 Bergstrom

¦ § yz ·§ yz ·
¦ (nx2  yz) ª(n  1) x2  yz º t
¨ 2  n ¸¨ 2  n  1¸ ¬ ¼
©x ¹© x ¹
Bergstrom
t
¦ x 2 2
¦x 4

t
 2¦ y 2 z 2 (1)

n(n  1)¦ x 4  (2n  1)¦ x 2 yz  ¦ y 2 z 2 n(n  1)¦ x 4  (2n  1)¦ x 2 yz  ¦ y 2 z 2

M d , unde (1) œ 2  2n2 ¦ x4  2n2  6n  1 ¦ y 2 z 2 t (6n  3) xyz ¦ x , adevărată din


(1)
3
t 2
n  3n  2
nd1
¦ x t ¦ y z Ÿ 2  2n ¦ x  2n  6n  1 ¦ y z t 2  2n ¦ y z 
4 2 2 2 4 2 2 2 2 2 2

 2n  6n  1 ¦ y z 2  2n  2n  6n  1 ¦ y z 6n  3 ¦ y z t (6n  3)¦ x yz
2 2 2 2 2 2 2 2 2 2

(6n  3) xyz ¦ x , evident din A  B  C t AB  BC  CA, cu A yz, B zx, C xy . Egalitatea are


2 2 2

loc pentru a b c 1.
1 3
2. Dacă a, b, c ! 0 astfel încât abc 1 demonstrați că ¦ (a  1)(a  n) t 2(n  1) , unde 0 d n d 2 .
yz zx xy 1
Soluție: Facem substituția a
x2
,b
y2
,c
z2
și obținem M s ¦ (a  1)(a  n)
¦ x2
2
1 x4 Bergstrom

¦ § yz ·§ yz · ¦ yz  x 2 yz  nx2 t
¨ 2  1¸¨ 2  n ¸
¦ yz  x 2 yz  nx 2
©x ¹© x ¹
¦ x  2¦ y 2 z 2
4 (1)
t
3
M d , unde
n¦ x  (n  1)¦ x yz  ¦ y z 2(n  1)
4 2 2 2
- ARTICOLE ȘI NOTE MATEMATICE -

(1) œ (2n  2)¦ x4  (4n  4)¦ y 2 z 2 t 3n¦ x 4  (3n  3)¦ x 2 yz  3¦ y 2 z 2 œ (2  n)¦ x 4 


(4n  1)¦ y 2 z 2 t (3n  3)¦ x 2 yz . Avem ¦ x t ¦ y z Ÿ (2  n)¦ x t (2  n)¦ y z
4 2 2 4 2 2
Ÿ

Ÿ 2  n ¦ x  4n  1 ¦ y z t 2  n ¦ y z  4n  1 ¦ y z 3n  3 ¦ y z t
4 2 2 2 2 2 2 2 2

t (3n  3)¦ x yz œ ¦ y z t ¦ x yz , adevărată din A  B  C t AB  BC  CA, cu


2 2 2 2 2 2 2

A yz, B zx, C xy . Egalitatea are loc pentru a b c 1 .


1 3
3. Dacă a, b, c ! 0 astfel încât abc 1 demonstrați că ¦ (a  n)(a  k ) t (n  1)(k  1) , unde
n, k t 0 și 2nk d n  k  1 .
yz zx xy 1
Soluție: Facem substituția a
x2
,b
y2
,c
z2
și obținem M s ¦ (a  n)(a  k )
¦ x2
2
1 x4 Bergstrom

¦ § yz ·§ yz · ¦ yz  nx2 yz  kx 2 t yz  nx 2 yz  kx 2
¨ 2  n ¸¨ 2  k ¸
¦
©x ¹© x ¹
¦ x  2¦ y 2 z 2
4 (1)
t
3
M d , unde
nk ¦ x  (n  k )¦ x yz  ¦ y z nk  n  k  1
4 2 2 2

(1) œ (n  k  1  2nk )¦ x4  (2nk  2n  2k 1)¦ y 2 z 2 t (3n  3k )¦ x 2 yz , care rezultă din


¦x t ¦ y z
4 2 2
Ÿ (n  k  1  2nk )¦ x 4  (2nk  2n  2k  1)¦ y 2 z 2 t
t0

Ÿ n  k  1  2nk ¦ y 2 z 2  2nk  2n  2k  1 ¦ y 2 z 2 t 2  n ¦ y 2 z 2  4n  1 ¦ y 2 z 2
3n  3k ¦ y 2 z 2 t (3n  3k )¦ x2 yz œ ¦ y 2 z 2 t ¦ x2 yz , adevărată din
A2  B2  C 2 t AB  BC  CA, cu A yz, B zx, C xy . Egalitatea are loc pentru
A B C, x y z Ÿ a b c 1 .
Bibliografie:
Art of Problems Solving 7/2016.
Marin Chirciu, Inegalități algebrice, de la inițiere la performanță, Editura Paralela 45, Pitești, 2014.

O metodă de rafinare a unei inegalităţi


de Titu Zvonaru, Comăneşti şi Neculai Stanciu, Buzău

Fie o inegalitate de forma MS t MD . Folosind o inegalitate cunoscută, obţinem o minorare MS t E şi


MD t E ; scriem apoi inegalitatea de la care am plecat sub forma MS  E t MD  E .
În demonstraţia acestei inegalităţi (de cele mai multe ori prin spargere), ajungem la ceva de
forma A(a  b) 2 t B(a  b) 2 . Pentru a z b , rămâne de arătat că A t B . Dacă această inegalitate este prea
slabă, atunci putem încerca să obţinem o rafinare a inegalităţii iniţiale.
Prezentăm în continuare rafinarea unor inegalităţi folosind ideea de mai sus.

1. Fie inegalitatea
¦a 3

t
¦ a , unde a, b, c ! 0 .
2

3abc ¦ ab
1
Cineva care ştie foarte bine inegalităţi, o poate demonstra aşa: folosim Cebâşev şi cunoscuta ¦a ˜¦ a t 9 ,
- ARTICOLE ȘI NOTE MATEMATICE -

¦a t
3
1 1
˜ ˜ ¦a ˜ ¦a2 t
1
˜ ¦a2 ˜
3 ¦a , 2

q.e.d.
3abc 3abc 3 3abc
¦a
1 ¦ ab
Un rezolvitor care are mai puţine cunoştinţe încearcă să găsească o cale de abordare;
Cu inegalitatea mediilor avem ¦
a 3 t 3abc , dar mai ştim că ¦ ¦
a 2 t ab , deci nu putem intercala nimic
între partea stângă şi partea dreaptă. Scriem atunci
¦a 3

t
¦ a ¦ ab œ ¦ a ˜ ¦ (a  b) t ¦ (a  b) .
 3abc 2 2 2

3abc ¦ ab 6abc 2¦ ab
¦ a (a  b) (a  b) 2 2
Acum observăm că este suficient să demonstrăm că t , (1).
6abc 2¦ ab

Dacă a b avem egalitate; dacă nu, rămâne de arătat că


¦a t 1 œ
6abc 2¦ ab
1
œ ¦ a ˜ ¦ ab t 3abc œ ¦ a ˜ ¦ t 3 , (2), care este adevărată, dar prea slabă (căci avem de fapt
a
1
¦ a ˜ ¦ a t 9 ). Acum intervine rafinarea: putem adăuga ceva în partea dreaptă a inegalităţii (1) astfel încât
inegalitatea (2) să rămână adevărată. Inegalitatea (1) poate fi înlocuită cu
( ¦ a ) ˜ ( a  b) 2
( a  b)2
2(a  b) 2

6abc
t
2¦ ab

2¦ ab
, care se reduce la ¦ a ˜ ¦ ab t 9abc , adevărată. Revenind,

avem de fapt următoarea rafinare a inegalităţii de la care am pornit


¦a 3

t
¦ a  ¦ ( a  b)
2 2

, (*).
3abc ¦ ab ¦ ab
Atunci când inegalitatea care joacă rolul lui (2) e prea strânsă, nu putem obţine o rafinare.
a3  b3  c3 § a b c ·
2. Dacă a, b, c ! 0 , atunci t 2¨   ¸.
abc ©bc ca a b¹
a3  b3  c3
Aplicând inegalitatea mediilor, avem t 3.
abc
a b c 3
Deoarece   t (inegalitatea lui Nesbitt-Ionescu), nu putem intercala ceva între partea
bc ca ab 2
stângă şi partea dreaptă. Scriem inegalitatea sub forma
a3  b3  c3 § a b c 3·
3t 2¨    ¸œ
abc ©bc c  a a b 2¹
(a  b  c)(a  b  c 2  ab  bc  ca )
2 2
§ a 1·
œ t 2¦ ¨  ¸œ
abc ©bc 2¹

œ ( a  b  c) ˜
¦ ( a  b) 2


( a  b) 2
.
2abc (a  c)(b  c)
( a  b) 2 ( a  b) 2
Se observă că este suficient să demonstrăm că (a  b  c) ˜ t , (3)
2abc (a  c)(b  c)
abc 1
Dacă a b , avem egalitate; dacă a z b , rămâne de arătat că t
2abc (a  c)(b  c)
œ (a  b  c)(a  c)(b  c) t 2abc œ (a  b  c)(c  ab  bc  ca) t 2abc , (4) care este adevărată
2

deoarece, aplicând inegalitatea mediilor, obţinem (a  b  c)(ab  bc  ca) t 9abc .


Rafinarea inegalităţii din enunţul problemei: deoarece avem de fapt
- ARTICOLE ȘI NOTE MATEMATICE -

¦ a ˜ ¦ ab  c ¦ a t 9abc  c ¦ a , putem aduna ceva în partea dreaptă a inegalităţii (3) astfel încât
2 2

(4) să rămână adevărată. Avem două posibilităţi:


Rafinarea 1. Daca adăugam pe 7abc , atunci se obţine rafinarea
¦a 3
a
t 2¦  7¦
( a  b) 2
.
abc bc 2(a  c)(b  c)

Demonstrarea acestei rafinări se reduce la a arăta că


¦a t 1

7
œ
2abc (a  c)(b  c) 2(a  c)(b  c)
œ ¦ a ˜ ¦ ab  c 2 ¦ a t 2abc  7abc , adevărată.
Rafinarea 2. Dacă adăugăm pe c 2 ¦ a , obţinem rafinarea
¦a 3
a
t 2¦ ¦
c(a  b  c)(a  b) 2
,
abc bc 2ab(a  c)(b  c)

a cărei demonstraţie se reduce la arăta că


¦a t 1

c(a  b  c)
œ
2abc (a  c)(b  c) 2ab(a  c)(b  c)
œ ¦ a¦ ab  c 2 ¦ a t 2abc  c 2 ¦ a , adevărată.
4¦ x 2 x 13
3. ¦ t , x, y, z ! 0 .
27¦ xy 7 x  y  z 27
¦ x t 1 şi
2
x 1
Deoarece ¦ d (se poate demonstra, de exemplu, cu substituţiile
¦ xy 7x  y  z 3
7x  y  z 9a, x  7 y  z 9b , x  y  7 z 9c ), scriem inegalitatea dată sub forma
4¦ x 2
4 1 x
MS  t MD ¦ .
27¦ xy 27 3 7x  y  z
4( x 2  y 2  z 2 ) 4 4( x 2  y 2  z 2 )  4( xy  yz  zx)
MS 
27( xy  yz  zx) 27 27( xy  yz  zx)

2 ( x  y )  ( y  z )  ( z  x)
2 2 2
.

27( xy  yz  zx)
1 x §1 x · § yx zx ·
MD ¦ ¦ ¨¨  ¸¸ ¦ ¨¨  ¸¸
3 7x  y  z © 9 7 x  y  z ¹ © 9( 7 x  y  z ) 9( 7 x  y  z ) ¹
yx zx yx x y
¦ 9(7 x  y  z)  ¦ 9(7 x  y  z) ¦ 9(7 x  y  z)  ¦ 9( x  7 y  z)
x  y§ 1 1 · 2( x  y)2
¦ 9 ¨¨ x  7 y  z  7 x  y  z ¸¸ ¦ 3(7 x  y  z)( x  7 y  z) .
© ¹
2( x  y ) 2 2( x  y) 2
Este suficient să demonstrăm că t .
27( xy  yz  zx) 3(7 x  y  z )( x  7 y  z )
Dacă x y , avem egalitate; dacă x z y rămâne să arătm că
1 1
t œ
27( xy  yz  zx) 3(7 x  7 y  z  50 xy  8 yz  8 zx)
2 2 2

œ 7 x 2  7 y 2  z 2  50 xy  8 yz  8zx t 9( xy  yz  zx) ,(5), adevărată, deoarece


x 2  y 2  z 2 t xy  yz  zx . Avem egalitate dacă şi numai dacă x y z .
- ARTICOLE ȘI NOTE MATEMATICE -

Putem aduna în partea dreaptă a inegalităţii (5) expresia 6( x  y) 2 şi


obţinem următoarea rafinare a inegalităţii din enunţ
4¦ x 2 x 13 4( x 2  y 2 ) 2
¦ t ¦ .
27¦ xy 7 x  y  z 27 9(¦ xy )(7 x  y  z )( x  7 y  z )
Pentru a demonstra această rafinare trebui să arătăm de fapt că
2( x  y) 2 2( x  y) 2 4( x  y) 2
t 
27¦ xy 3(7 x  y  z )( x  7 y  z ) 9(¦ xy )(7 x  y  z )( x  7 y  z )
œ 7 x 2  7 y 2  z 2  50 xy  8 yz  8xz t 9¦ xy  6 x 2  6 y 2  12 xy ,
adevărată, deoarece ¦x 2
 8¦ xy t 9¦ xy .

¦ a t 3¦ a . 2 2

4. Dacă a, b, c ! 0 , atunci
¦ ab ¦ a
Deoarece ¦ a t ¦ ab şi 3 ˜ ¦ a t ¦ a , scriem inegalitatea din enunţ sub forma
2 2

¦ a  ¦ ab t 3¦ a  ¦ a œ œ ¦ a  ¦ ab t 3¦ a  ¦ a œ
2 2
2 2 2 2

¦ ab ¦a ¦ ab ¦ a 3¦ a  ¦ a 2

( a  b) 2 ( a  b) 2
œ¦ t¦
2¦ ab ¦ a ˜ 3 ¦ ¦a
2
 a
.

( a  b) 2 ( a  b) 2
Observăm că este suficient să demonstrăm că
2¦ ab
t
¦a ˜ 3¦ a 2
 ¦a .
Dacă a b avem egalitate; dacă nu rămâne de arătat că
œ ¦ a  ¦ a ˜ 3¦ a 2 t 2¦ ab , (6).
1 1 2
t
2¦ ab ¦ a ˜ 3¦ a  ¦ a2
œ

Ultima inegalitate este adevărată deoarece ¦ a t 3¦ ab şi 2

¦a ˜ 3¦ a t ¦ a ˜ ¦ a ¦ a .
2 2

Cum inegalitatea (6) este prea slabă, putem obţine următoarea rafinare a inegalităţii iniţiale:

¦ a t 3¦ a
2 2
( a  b) 2
 2¦
¦ ab ¦ a ¦ a ˜ ¦ a  3¦ a 2
, (7).
Ținând cont de cele de mai sus, demonstrarea inegalităţii (7) se reduce la a arăta că
1 1 2
¦ a ˜ ¦ a  3¦ a ¦ a ˜ ¦ a  3¦ a
t  œ
2¦ ab 2 2

œ ¦ a ˜ ¦ a  3¦ a t 2¦ ab  4¦ ab , adevărată (conform celor de mai sus).


2

a2 b2 c2 2 a
5. Dacă a, b, c sunt laturile unui triunghi, atunci   t ˜ ¦a ˜ ¦ .
b c a 3 bc
a2 b2 c2 ¦ a 2

Folosind inegalitatea lui Bergström obţinem   t ¦a .


b c a ¦a
Pe de altă parte, datorită inegalităţii lui Nesbitt-Ionescu avem
2 a 2 3
˜ ¦a ˜ ¦ t ˜ ¦a ˜ ¦a .
3 bc 3 2
Scriem inegalitatea din enunţ sub forma
- ARTICOLE ȘI NOTE MATEMATICE -

a2 b2 c2 2 § a 3·
 b  2a   c  2b   a  2c t ˜ ¦ a ˜ ¨ ¦  ¸œ
b c a 3 © bc 2¹
( a  b) 2
1 ( a  b) 2
œ¦ t ¦ a¦ .
b 3 (a  c)(b  c)
( a  b) 2 ( a  b) 2
Este suficient să demonstrăm că t ¦a ˜ .
b 3(a  c)(b  c)

Dacă a b avem egalitate; dacă nu, rămâne de arătat că


1
t
¦a œ
b 3(a  c)(b  c)
œ 3ab  3bc  3ac  3c 2 t ab  b 2  bc , (8),
œ b(a  c  b)  ¦ ab  3c 2  2ac t 0 , adevărat deoarece a, b, c sunt laturile unui triunghi.
Deoarece inegalitatea (8) este slabă, putem obţine următoarea rafinare a inegalităţii din enunt
a2 b2 c2 2 a ( a  b) 2
  t ˜ ¦a ˜ ¦  ¦ ab ˜ ¦ , (9).
b c a 3 bc 3b(a  c)(b  c)
1
Demonstrarea inegalităţii (9) se reduce la a arăta că t
¦a 
¦ ab
b 3(a  c)(b  c) 3b(a  c)(b  c)
œ b(a  c  b)  3c  2ac t 0 , adevărată.
2

Propunem ca exerciţiu demonstrarea rafinării


a2 b2 c2 2 a ( a  b) 2
  t ˜ ¦a ˜ ¦ ¦ , (10)
b c a 3 bc 3b
şi de asemenea lăsăm cititorului posibilitatea de a obţine alte rafinari ale inegalităţilor de la care am pornit.

Integrale duble
prof. Adrian Stan, Buzău

Materialul de față prezintă o serie de metode de calcul al integralelor duble, integrale care se
calculează din funcții f(x;y) mărginite și definite pe un domeniu mărginit D  2 u .
Să considerăm o partiție a planului în intervale bidimensionale, din care alegem pe cele care conțin
puncte din D. Notăm prin Zk , k 1, 2,3,..., n, intervalele bidimensionale alese, numerotate într-o ordine
arbitrară. Fie diviziunea ' mulțimea acestor intervale. Norma diviziunii ' , notată ' , este cea mai mare
dintre dimensiunile intervalelor Zk , k 1, 2,3,..., n.
Z1 > x1; x2 @ u > y1; y2 @ , Z2 > x2 ; x3 @ u > y2 ; y3 @ , etc.
Suma Riemann atașată funcției f(x;y), corespunzătoare diviziunii ' a domeniului D, este
n
V'( f ) ¦ f (c , n ) ˜ aria Z ,
k 1
k k k (ck ; nk )  Zk , unde (ck , nk ) sunt un sistem de puncte intermediare.
Atunci, integrala dublă a funcției f(x;y) extinsă la domeniul D este

³³ f ( x; y)dxdy
D
limV ' ( f ) , pentru orice alegere a sistemului de puncte intermediare (ck , nk ) .
' o0

Pentru calculul unei integrale duble există o serie de tipuri fundamentale de domenii de integrare:
1. Dacă D este dreptunghiul > a; b@ u >c; d @ , atunci
b
§d · d
§b ·
³³ f ( x; y)dxdy
D
³a ¨© ³c f ( x; y)dy ¸¹dx ³c ¨© ³a f ( x; y)dx ¸¹dy .
- ARTICOLE ȘI NOTE MATEMATICE -

2. Dacă D este un domeniu închis, simplu în raport cu axa OY,


D ^ x; y  2
a d x d y, D ( x) d y d E ( x)` , iar funcția f ( x; y) este integrabilă pe >D ( x); E ( x)@ , atunci
b § E ( x) ·
³³ f ( x; y)dxdy ³a ¨ D ³( x )
¨ f ( x; y ) dx ¸dy.
¸
D © ¹
Mai există de asemenea și situațiile când D este un domeniu închis, simplu în raport cu axa OX sau
când este nevoie să trecem de la coordonate carteziene la coordonate polare.
În concluzie, dacă D este un domeniu închis și mărginit, atunci aria suprafeței D este egal cu
Aria( D) ³³ dxdy .
D
Câteva exemple de calcul:
1) Fie D >0;1@ u >0;1@ și f : D o , f ( x; y) 4 x2 y  3xy 2  1 . Să se calculeze ³³ f ( x; y)dxdy .
D
Rezolvare:
1
ª1 º 1
ª 2 y2 y3 y 1º 1

³0 «¬ ³0   ³0 «¬   ³ (2x  x  1)dx
2 2 2
I (4 x y 3 xy 1) dy »dx (4 x 3 x y ) »dx
¼ 2 3 y 0¼ 0

§ x x ·1
3 2
2 1 7
¨2   x¸  1 .
© 3 2 ¹0 3 2 6

2. Fie D ^ x; y  2
`
y d x  3, y t x 2  2 x  3 . Să se calculeze ³³ dxdy .
D

Rezolvare: Vom considera funcțiile f1 , f 2 : o , f1 ( x) x  2 x  3, f 2 ( x) x  3 .


2

Să determinăm punctele de intersecție ale celor două grafice:


f1 ( x) f 2 ( x) Ÿ x 2  2 x  3 x  3
Ÿ x2  3x 0 Ÿ x( x  3) 0 Ÿ x1 0, x2 3 Ÿ y1 3, y2 0.
Așadar, punctele de intersecție a graficelor celor două funcții sunt
A(0; 3), B(3;0).
Domeniul D este dat de suprafața hașurată, rezultă că D se mai
poate scrie de forma
D ^ x; y  2
0 d x d 3, x 2  2 x  3 d y d x  3 , rezultă `
că D e simplu în raporrt cu axa OY, și atunci
3
ª x 3 º 3
ª y x 3 º
I ³0 « 2 ³ »dx
« dy » ³0 « y y
« »
x 2  2 x  3¼»
dx
¬ x  2 x 3 ¼ ¬
3
§ x3 x2 · 3
³0 ( x  3x)dx ¨©  3  3 2 ¸¹ 0
2

27 27 27 9
 9  .
3 2 2 2
3. Să se calculeze I ³³ dxdy, unde D este domeniul dat de
D
suprafața hașurată.
Rezolvare:
Mai întâi se determină ecuațiile dreptelor AB și BC:
- ARTICOLE ȘI NOTE MATEMATICE -

x y 1 x y 1
AB: 0 2 1 0Ÿ y x  2 și BC: 1 3 1 0 Ÿ y 5  2 x . Vom integra pe domenii simple în
1 3 1 2 1 1
raport cu OY. Domeniul D se descompune în reuniunea a două domenii D1, D2, care au interioare disjuncte.
1
§ x2 ·
Pentru D1: a 0, b 1 , D ( x) 0, E ( x) x  2 . Atunci, I1 ³³ dxdy
D
³0 ¨© ³0 dy ¸¹dx
1
§ y x  2· 1
§ x2 ·1 1 5
³0 © y y
¨ ³0
¸dx
x  2 dx ¨  2 x ¸  2 .
0 ¹ © 2 ¹ 0 2 2
Pentru D2: a 1, b 2 , D ( x) 0, E ( x) 5  2 x . Atunci,
2 5 2 x
§ · 2
§ y 5  2x · 2
2
I 2 ³³ dxdy ³ ¨ ³ dy ¸dx ³ ¨ y
y 0
¸dx ³ (5  2 x)dx 5x  x 1
2
2.
D 1© 0 ¹ 1© ¹ 1
5 9
Așadar, I I1  I 2 2 .
2 2
4. Să se calculeze aria domeniului D, unde D este limitat de dreptele de ecuații x  y 2,
y 3x  2, x 3 y  2.
Rezolvare: Se găsesc mai întâi punctele de intersecție ale dreptelor: Astfel, A(0;2), B(-1;-1), C(2;0).
Domeniul dat se împarte în două domenii, astfel că AD A1  A2 .
§ 3x2 · § y 3x  2 ·
x2·
0 0 0
¨ ¸ ¨ ¸ §
I1 ³³ dxdy ³1 ¨ x³2 dy ¸dx ³1 ¨¨ y y x  2 ¸dx
¸
³ ¨© 3x  2  ¸dx
3 ¹
D1 ¨ ¸ © 3 ¹
1
© 3 ¹
x2 1 x2 2 x 0 4
(3  2x  ˜  ) .
2 3 2 3 1 3
§ 2 x · § y 2 x ·
x2·
2 2 2
¨ ¸ ¨y ¸ § 2
x2
I2 ³³ dxdy ³0 ¨ x³2 dy ¸dx ³0 ¨¨ y x  2 ¸dx
¸
³0 ©
¨ 2  x  ¸dx
3 ¹ ³ (2  x  )dx .
D2 ¨ ¸ © 3 ¹ 0
3
© 3 ¹
§ x2 1 x2 ·2 2 4 8
¨ 2 x   (  2 x) ¸ 42  .
© 2 3 2 ¹0 3 3 3
4 8
Așadar, I ³³ dxdy
D
I1  I 2 
3 3
4.

Altfel, dacă se cunosc coordonatele vârfurilor triunghiului ABC,


se poate folosi formula pentru arie cu ajutorul determinanților .
5. Să se calculeze momentele de inerție în raport cu axele de
coordonate și în raport cu originea pentru placa (D) definită
prin x  y d 1, x t 0, y t 0, știind că densitatea de masă
este f ( x; y) xy .
Rezolvare: Mai întâi se determină domeniul D care este
triunghiul OAB, cu A(1;0), B(0;1). Atunci, momentele de inerție
în raport cu axele OX, resp. OY sunt conform formulelor:
1
§ 1 y 3 · 1
ª§ x 2 3 · x 1  y º
³³ y ˜ f ( x; y)dxdy ³0 ¨¨ ³0 xy dx ¸¸dy ³0 «¬¨© 2 ˜ y ¸¹ x 0 »¼dy
2
Ix
D © ¹
- ARTICOLE ȘI NOTE MATEMATICE -

1 1
˜ ³ y 3  2 y 4  y 5 dy
1 1 1
˜ ³ (1  y)2 ˜ y 3dy .
2 0 2 0 120
1
§ 1 x 3 · 1
ª§ y 2 3 · y 1  x º
³³ x ˜ f ( x; y)dxdy ³0 ¨© ³0 x ydy ¸¹dx ³0 «¬¨© 2 ˜ x ¸¹ y 0 »¼dx
2
Iy
D
1 1
˜ x3  2 x 4  x5 dy
1 1 1
2 ³0 2 ³0
˜ (1  x) 2 ˜ x3dx . Momentul de inerție al plăcii în raport cu originea
120
1 1 1
este dată de formula I0 Ix  I y  .
120 120 60
Integralele duble se mai pot folosi la calculul volumului (V) unui corp delimitat de o suprafață f(x;y) sau
la determinarea masei (M) unei plăci plane D, de densitate f ( x; y)² 0 cu ajutorul formulei
V ³³ f ( x; y)dxdy , M ³³ f ( x; y)dxdy sau la determinarea coordonatelor centrului de greutate G al
D D
1
unei plăci plane D, cu densitatea de masă f ( x; y)² 0 : xG
M ³³ x ˜ f ( x; y)dxdy ,
D
1
yG
M ³³ y ˜ f ( x; y)dxdy .
D
Bibliografie:
1. Probleme de matematici superioare. S. Chiriță. Editura Didactică și Pedagogică. București. 1989.
2. Matematici aplicate în economie. Silvia Dedu, Florentin Șerban. Editua Teocora. 2009.
Prof. Liceul Tehnologic Costin Nenițescu, Buzău

Proprietati ale patrulaterului convex - Aplicatii


de Soare Carmen Violeta, București

1. Fie ABCD un patrulater convex în planul P. Să se determine mulţimea punctelor M din planul P cu
proprietatea că MA2+MC2= MB2+MD2. Discuţie.
Notăm cu E, respectiv F, mijloacele segmentelor
(AC) şi (BD). Din teorema medianei rezultă că : B
1
MA2+MC2=2 ME2+ AC2 şi MB2+MD2 =
2 A
1
2 MF + BD2. Egalitatea din enunţ este
2
F
2 E
echivalentă cu:
1 1
2 ME2+ AC2 =2 MF2+ BD2 sau
2 2 D C
1
ME2-MF2= (BD2-AC2) = const.
4 M
Distingem următoarele cazuri:
a. E z F şi atunci mulţimea este o dreaptă perpendiculară pe (EF) (acest caz are loc dacă patrulaterul ABCD nu
este paralelogram).
b. E=F şi (BD) { (AC) şi atunci mulţimea este tot planul P (în acest caz patrulaterul ABCD este dreptunghi).
c. E=F şi BD z AC şi atunci mulţimea este vidă (acest caz are loc dacă patrulaterul ABCD este paralelogram,
dar nu este dreptunghi).

2. Fie cinci puncte distincte A, B, C, D, E în plan astfel încât ABCD şi ABDE sunt patrulatere convexe.
Să se arate că ABCD este patrulater convex.
Deoarece ABCD şi ABDE sunt patrulatere convexe rezultă că (AC) ∩(BD)={M} şi (AD) ∩(BE)={N}.
- ARTICOLE ȘI NOTE MATEMATICE -

Dreapta AM separă B şi N; fie AM∩(BN)={P}. Analog dreapta


BN separă A şi M; fie BN∩(AM)={Q}.
A
Rezultă că (AM) şi (BN) au un punct comun,
acesta fiind P=Q. Deoarece fie
AM∩(BN)={P}, rezultă fie P B
AC ∩(BE) z Ø, deci ABCE este patrulater
convex. (fig de mai sus) E

3. Să se arate că punctul din plan pentru N


M
care suma distanţelor la vârfurile unui
patrulater convex este minimă, este
intersecţia diagonalelor patrulaterului.
Fie patrulaterul convex ABCD şi fie C
D
{O}=[AC]∩[BD], iar M  P. Deosebim cazurile
A
cazurile când M aparţine cel puţin unei diagonale şi
B când M aparţine cel puţin unei diagonale şi când M
O nu aparţine cel puţin unei diagonale şi când M nu
aparţine nici unei diagonale. Oricum, însă putem
scrie: MA+MC t AC şi MB+MD t BD şi deci
MA+MB+MC+MD t AC+BD = OA+OB+OC+OD.
4. Fie ABCD un patrulater convex. Notăm
C
AB∩CD)={E}, BC∩AD={F}. Să se arate că
mijloacele M, N, P ale segmentelor [AC],
D [BD][EF] sunt coliniare. (teorema NEWTON-
GAUSS).
M A

Fie G,H,I mijloacele segmentelor [CE], [EB], [BC].


Punctele G, I, M sunt pe o paralelă la EAB; punctele H, I,
N sunt pe o paralelă la ECD, iar punctele H, G, P sunt pe o M
paralelă la BCF.
Deci M, N, P sunt pe prelungirile laturilor GHI. B D
Conform teoremei lui Menelaos coliniaritatea punctelor M,
N, P este echivalentă cu îndeplinirea egalităţii: N
MI GP HN
∙ ∙ =1
MG PH NI
Folosind linii mijlocii convenabile vom putea scrie: H C
AB CF ED
MI A B GP CF HNI ED
= 2 = ; = 2 = ; = 2 = ; G F
MG AE AE PH BF BF NI CD CD
2 2 2 E P
AB CF ED
Egalitatea de mai sus devine ∙ ∙ =1, ea constituind
AE BF CD
relaţia lui Menelaos pentru triunghiul BEC şi punctele coliniare A, D, F. Această teoremă datorată lui ISAAC NEWTON
din 1687 a fost completată de KARL FRIEDRICH (1777-1855). Matematician, fizician şi astronom german, profesor
universitar la GÖTINGEN, GAUSS este autorul unor lucrări în domeniile algebrei, teoriei numerelor, analizei
matematice, geometriei diferenţiale, studiul planetelor. El a rezolvat problema construcţiei cu rigla şi compasul a
poligoanelor regulate şi a definit curbura totală. Deşi a publicat puţin, GAUSS este exemplar sub aspectul profunzimii
rezultatelor sale. El a scos în evidenţă deficienţe ale raţionamentelor matematicienilor secolului al XVIII-lea, pe care le-a
refăcut.
Bibliografie:
1. Mihalca , D., Chițescu I., Chiriță M., Geometria patrulaterului, Ed. Teora, Bucureşti, 1998.
2. Mica enciclopedie matematică, Ed. Tehnică, Bucureşti, 1980.
Prof., Școala Gimnazială ,, George Bacovia”, București
- ARTICOLE ȘI NOTE MATEMATICE -

Asupra unor sume de progresii aritmetice


Ciobîcă Constantin, Colegiul Vasile Lovinescu,Fălticeni
Ciobîcă Elena , Colegiul Mihai Băcescu,Fălticeni

În cele ce urmează vom considera un șir an nt1 de numere reale în progresie aritmetică de raţie r
pentru care vom demonstra următoarele relații:
1).
1 1 1 n 1
  ...  .
a3  a5 ˜ a7  a9 a7  a9 ˜ a11  a13 a2n1  a2n3 ˜ a2n5  a2n7 2 ˜ a3  a5 ˜ a2n5  a2 n7
2).
1 1 1 n 1
  ...  .
a7  a11 ˜ a15  a19 a15  a19 ˜ a23  a27 a4n3  a4n7 ˜ a4n11  a4n15 2 ˜ a7  a11 ˜ a4n11  a4n15
3)
1 1 1 n 1
  ...  .
a11  a17 ˜ a23  a29 a23  a29 ˜ a35  a41 a6n5  a6n11 ˜ a6n17  a6n23 2 ˜ a11  a17 ˜ a6n17  a6n23
Rezolvare:
1). a3  a5 a1  2r  a1  4r 2a1  6r și a7  a9 a1  6r  a1  8r 2a1  14r
1 1 § 1 1 ·
a7  a9  a3  a5 8r Ÿ ˜ ¨¨  ¸
a3  a5 ˜ a7  a9 8r © a3  a5 a7  a9 ¸¹
a11  a13 a1  10r  a1  12r 2a1  22r
1 1 § 1 1 ·
a11  a13  a7  a9 8r Ÿ ˜ ¨¨  ¸
a7  a9 ˜ a11  a13 8r © a7  a9 a11  a13 ¸¹
a2n1  a2n3 a1  2nr  a1  2n  2 r 2a1  4n  2 r
a2n5  a2n7 a1  2n  4 r  a1  2n  6 r 2a1  4n  10 r
a2n5  a2n7  a2n1  a2n3 8r
1 1 § 1 1 ·
Ÿ ˜ ¨¨  ¸
a2n1  a2n3 ˜ a2n5  a2n7 8r © a2 n1  a2 n3 a2 n5  a2 n7 ¸¹
1 § 1 1 · 1 a2 n5  a2 n7  a3  a5
ŸS ˜ ¨¨  ¸ ˜
8r © a3  a5 a2 n5  a2 n7 ¸¹ 8r a3  a5 ˜ a2 n5  a2 n7
a2n5  a2n7  a3  a5 2a1  4n  10 r  2a1  6r 4n  4 r 4 n  1 r
4 n  1 r n 1
S .
8r ˜ a3  a5 ˜ a2 n5  a2 n7 2 ˜ a3  a5 ˜ a2 n5  a2 n7

2) a7  a11 a1  6r  a1  10r 2a1  16r și a15  a19 a1  14r  a1  18r 2a1  32r
1 1 § 1 1 ·
a15  a19  a7  a11 16r atunci, ˜ ¨¨  ¸
a7  a11 ˜ a15  a19 16r © a7  a11 a15  a19 ¸¹
a23  a27 a1  22r  a1  26r 2a1  48r și a23  a27  a15  a19 16r
1 1 § 1 1 ·
˜ ¨¨  ¸
a15  a19 ˜ a23  a27 16r © a15  a19 a23  a27 ¸¹
- ARTICOLE ȘI NOTE MATEMATICE -

a4n3  a4n7 a1  4n  2 r  a1  4n  6 r 2a1  8n  8 r


a4n11  a4n15 a1  4n  10 r  a1  4n  14 r 2a1  8n  24 r
a4n11  a4n15  a4n3  a4n7 16r
1 1 § 1 1 ·
˜ ¨¨  ¸
a4n3  a4n7 ˜ a4n11  a4n15 16r © a4n3  a4n7 a4n11  a4n15 ¸¹
S
1 § 1
˜ ¨¨ 
1 ·
¸¸
a4n11  a4n15  a7  a11
16r © a7  a11 a4 n11  a4 n15 ¹ 16r ˜ a7  a11 ˜ a4 n11  a4 n15
a4n11  a4n15  a7  a11 8 ˜ n  1 ˜ r
8 n  1 r n 1
ŸS .
16r ˜ a7  a11 ˜ a4 n11  a4 n15 2 ˜ a7  a11 ˜ a4 n11  a4 n15

3) a11  a17 a1  10r  a1  16r 2a1  26r și a23  a29 a1  22r  a1  28r 2a1  50r
a23  a29  a11  a17 24r
1 1 § 1 1 ·
˜ ¨¨  ¸
a11  a17 ˜ a23  a29 24r © a11  a17 a23  a29 ¸¹
a35  a41 a1  34r  a1  40r 2a1  74r și a35  a41  a23  a29 24r
1 1 § 1 1 ·
˜ ¨¨  ¸
a23  a29 ˜ a35  a41 24r © a23  a29 a35  a41 ¸¹
a6n5  a6n11 a1  6n  4 r  a1  6n  10 r 2a1  12n  14 r
a6n17  a6n23 a1  6n  16 r  a1  6n  22 r 2a1  12n  38 r
a6n17  a6n23  a6n5  a6n11 24r
1 1 § 1 1 ·
˜ ¨¨  ¸
a6n5  a6n11 ˜ a6n17  a6n23 24r © a6 n5  a6 n11 a6 n17  a6 n 23 ¸¹
1 § 1 1 · 1 a6 n17  a6 n 23  a11  a17
S ˜ ¨¨  ¸ ˜
24r © a11  a17 a6 n17  a6 n 23 ¸¹ 24r a11  a17 ˜ a6 n17  a6 n23
a6n17  a6n23  a11  a17 12 ˜ n  1 r
1 12 n  1 r n 1
˜ .
24r a11  a17 ˜ a6 n17  a6 n23 2 ˜ a11  a17 ˜ a6 n17  a6 n23

Generalizations of the limits of the sequences of Bătineţu, Ghermănescu,


Ianculescu, Lalescu and other collaborations
D.M. Bătineţu-Giurgiu, Dan Sitaru and Neculai Stanciu

§ n  1 2t n 2t ·¸
I. If Bn (t ) n ¨
1t
 , with t ! 0 , then lim Bn (t ) te t .
¨
© n1 n  1 ! t
n
t ¸
n! ¹
n of

t
n 2t § n · un  1
Proof. We have Bn (t ) n 1t
˜ ˜ u n  1 ¨¨ n ¸¸ ˜ ˜ ln u nn , n t 2 , (1).
n!
n
t
© n! ¹ ln u n
- ARTICOLE ȘI NOTE MATEMATICE -

t
§ n  1 · §¨ n n! ·¸
2t

where we denoting u n ¨ ¸ ˜ , n t 2 . We have


© n ¹ ¨© n 1 (n  1)! ¸¹
u 1
lim u n 1 and then we obtain lim n 1 . We also
n of n of ln u
n
t
§ 2t § n! · ·¸ 2t
t
§ n1 (n  1)! ·
have: lim u n ¨
lim en ˜ ¨¨ ˜ (n  1)! ¸¸
n 1 e ˜ lim ¨ ¸ e 2 t ˜ e t et .
n of ¨  ¸ n of ¨  ¸
n
n of
© © ( n 1)! ¹ ¹ © n 1 ¹
By (1) and above we obtain that: lim Bn (t ) e ˜ 1 ˜ ln e
t t
te t
nof

§ (n  1) 2 n2 ·
Observation. For t 1 we obtain that: lim Bn (1) lim ¨ n ¸ e,
n of¨ n 1
n of
© (n  1)! n! ¸¹
i.e. the limit of well-known D.M Bătineţu-Giurgiu’ s sequence.
II. Let t  R * and the sequence I n (t ) nt2 , I n (t ) n1t §¨ n  1
©

t n 1
n  1  nt
t
n ·¸¹ , then
n
t

lim I n (t ) t
n of

Proof. We have that I n (t ) n ˜ n n t ˜ u n  1 n ˜ uln u 1 ˜ ln u


n
t
n n
n , n t 2 , (1).
n
t
§ n  1·
t
§ n 1 n  1 · un  1
where u n ¨ ¸ ˜ ¨¨ n ¸ , n t 2 . So, lim u n 1 , lim 1 . Also we have
© n ¹ © n ¸¹ n of n of ln u
n
nt t
§ n1 n  1 · § n 1 1 ·
lim u n
nlim e ˜ lim ¨¨ n
t
n
¸
¸ e ˜ lim ¨¨
t
˜ n 1 ¸¸ et ˜1 et .
n of n of n of
© n ¹
n of
© n n 1 ¹
Hence: lim I n (t ) 1 ˜ 1 ˜ ln lim u nn
nof

nof
ln e t t.
Observation. For t 1 we get that lim I n lim I n (1) 1 , i.e. the limit of the sequence of Romeo T.
nof nof
Ianculescu.

III. Let g n
(n  2) n 1
(n  1) n
2
, n  N , x  R . lim n sin x g ncos
nof
1
2
x 2

2
 g ncos x = cos 2 x ˜ e cos x
cos2 x

Proof. Gn ( x ) n 1cos2 x
g cos2 x
n 1 g cos2 x
n §g ·
n˜¨ n ¸
© n ¹
u n  1
cos2 x
(n  2) ( n1) cos
2
§ gn · un  1 x
un  1
¨ ¸ ln u nn 2 2
˜ ˜ ln u nn
© n ¹ ln u n n cos x (n  1) n cos x
ln u n
( n 1) cos2 x cos2 x
§n 2· § n 1· un  1
¨ ¸ ¨ ¸ ln u nn , n  N * , (1).
© n 1 ¹ © n ¹ ln u n
cos2 x cos2 x
§ g n 1 · § (n  3) n  2 (n  1) n ·
where u n ¨¨ ¸¸ ¨¨ ˜ ¸ , n  . Therefore,
© gn ¹ © (n  2)
n 1
(n  2) n 1 ¸¹
cos2 x
§ § n  3 · n2 § n  1 · n · § 1·
cos2 x
un  1
lim u n lim ¨ ¨ ¸ ¨ ¸ ¸¸ ¨e ˜ ¸ 1 , şi deci lim 1.
n of ¨ n  2 
n of
© © ¹ © n 2 ¹ ¹ © e¹ n of ln u n
n cos2 x
§ (n  3) (n  1) ·
n2 n n cos2 x
§ § (n  1)(n  3) · n 1 n  3 ·
lim u nn lim ¨¨ ˜ ¸ lim ¨ ¨¨ ¸ ˜ ¸
n of n of ( n  2) n 1
© (n  2) n 1 ¸¹ n of ¨
© (n  2) 2 ¸¹ n 1 ¸
© ¹
- ARTICOLE ȘI NOTE MATEMATICE -

( n 2  n ) cos2 x n cos2 x
§ n 2  4n  3 · § n  3·
e cos x ˜ e 2 cos
2 2 2
lim ¨¨ 2 ¸ ˜ lim ¨ ¸
x
e cos x .
n of n  4n  4 ¸ n of n  1
© ¹ © ¹
Hence we obtain that: lim Gn ( x)
nof
2
e cos x ˜ 1 ˜ 1 ˜ ln lim u nn nof
2
e cos x ˜ ln e cos
2
x 2
cos 2 x ˜ e cos x .
Observation. For x 0 , yields that lim Gn (0) lim Gn e , i.e. the limit of the sequence of Mihail
nof nof
Ghermănescu.

IV. Euler-Mascheroni-Lalescu collaboration


a  b ln c
If a, b  R , a  b 1 , then lim §¨ n  1 n 1 n  1 !cn  n a n n!en ·¸
a b b
, where
n of© ¹ eb
n
§ 1· n
1
en ¨1  ¸ and cn  ln n  ¦ .
© n¹ k 1 k

Proof. We have
Bn n  1 a n1 n  1 !cn b  n a n n!en
b
n a n n!en u n  1
b

n!en b
b
u 1 n
u 1 § n n!en · un  1
n an
n!en b
˜ n ˜ ln u n n˜ ˜ n ˜ ln u n ¨
¨ n
¸ ˜
¸ ln u n
˜ ln u nn , n t 2 , unde
ln u n nb ln u n © ¹
§ n 1 n  1 !cn
b
§ n  1·
a
·
un ¨ ¸ ¨ ¸ , n t 2 .
© n ¹ ¨ n n!e ¸
© n ¹
We have that
n n!cn n!cn § n  1 !cn 1 n n · cn1 1 1
lim lim n lim ¨¨ ˜ ¸
¸ n of c ˜ e
lim ;
n of n n of nn
n of
© n  1 n 1
n! c n ¹ n n e
n n!en n!e § n  1 !en 1 n · n
e 1 1
lim lim n n n lim ¨¨ ˜ ¸ lim n 1 ˜ .
n of n n of n n of
© n  1
n 1
n!en ¸¹ nof en en e
u 1
So lim u n 1 and then lim n 1 . Also we have that
n of n of ln u
n
b
§ n  1 !cn 1 · §c n 1 · §c ·
b

lim u n
lim e ¨ a
˜ ¸ e ˜ lim ¨ n ˜
a ¸ e ¨ ˜ e¸
a
eacb .
¨ n!en n 1 n  1 !c ¸ n of ¨ e n 1 n  1 !c ¸
n n
n of n of
© n ¹ © n n ¹ ©e ¹


b
a  b ln c
Hence: lim Bn
§1·
¨ ¸ ˜ 1 ˜ ln lim u n
n 1

˜ ln e a c b .
n of
© ¹
e n of eb eb
V. Euler-Mascheroni-Bătineţu collaboration
n
§ 1· n
1
Let en ¨1  ¸ and J n  ln n  ¦ , for any positive integer n , then
© n¹ k 1 k

§
lim ¨
n  1 2  n2 · e
¸ 2  ln J .
n of¨ n 1 ¸ 2
© 2 n  1 !!J n
n 2 n  1 !! e n ¹

Proof. We have xn
n  1 2 
n2 n2
u n  1
n 1 (2n  1)!!cn n (2n  1)!!en n (2n  1)!!en
- ARTICOLE ȘI NOTE MATEMATICE -

n un  1
˜ ˜ ln u nn , n t 2 , (1).
n (2n  1)!!en ln u n

§ n  1·
2 n
(2n  1)!!en
where we denoted u n ¨ ¸ , n t 2 .
© n ¹ n 1 (2n  1)!!cn
Also we have that:
n (2n  1)!!en (2n  1)!!en 1 § n · § 1 ·
n
§ 1 1 · 2
lim lim ¨ ¸ ¨ ¸ lim ¨¨ 2n  1 ˜ ˜ ¸¸ ,
n of (2n  1)!!e
n © n  1¹ © n 1¹ n  1 en
n of n n of e
© ¹
n 1 (2n  1)!!J n 2
and analogous, lim .
n of n 1 e
un  1
Yields that: lim u n 1 , and lim 1 . We have
n of n of ln u n
n 1 (2n  1)!!J
§ (2n  1)!!en n 1 · e3
lim u nn lim ¨¨ en2 ˜ ˜ (2n  1)!!J n ¸¸ lim n
n of n of
© ( 2 n  1 )!!J n ¹ J n of 2 n  1
n 1 ( 2n  1)!!J
e3 n  1 e3 2 1 e 2
lim n
˜ ˜ ˜ , (2).
J n of n 1 2n  1 J e 2 J
Hence, taking to limit in (1) with n o f and considering (2) we obtain that
e
lim xn
n of
2  ln J .
2

Euclid filozoful

Despre marele matematician Euclid (330-275 î.e.n) cel care


a întemiat celebra sa școală în Alexandria și de la care ne-a rămas
cartea ”Elementele ” , cea mai citită carte științifică, după Biblie,
ne-au rămas multe descoperiri în matematică dar și unele
întâmplări povestite de istorici cu mult timp după moartea sa.
Se poveștește că o dată, la școala sa, un elev după ce învățase
o serie de teoreme de geometrie l-a întrebat pe Euclid:
- Ce folos voi avea eu învățând aceste lucruri?
Auzind acestea, Euclid îl cheamă la el pe ajutorul său și-i spune:
- Dă-i acestuia trei monezi, căci el vrea să
câștige din ceea ce învață. Astfel a replicat marele învățat
considerând că unii dintre elevi vor să aibă un câștig imediat fără a
înțelege esența lucrurilor mai ales în domeniul matematicii unde
primele noțiuni te ajută să le înțelegi pe următoarele.
- PROBLEME REZOLVATE -

„ Acela care învață, dar nu gândește este pierdut;


cel care gândește, dar nu învață
se află într-un mare pericol”.
Confucius
(551- 479)

3. Probleme rezolvate

ƒ Clasa a V-a
G:732. Aflaţi n  , dacă 71 23.... n este pătrat perfect. Ion Stănescu, Buzău
Rezolvare:
n(n  1)
Evident 1  2  3  ...  n 2k , k  *,
*, Ÿ n(n  1) 4k . Așadar, n= 4k sau n= 4k-1.
2
G:733. Să se determine numerele naturale m și n știind că
m2 n  3  1  22  23  ...  22016 : ª«22017  20 º» 31214 .
2016
Iuliana Trașcă, Olt
¬ ¼
Rezolvare : 1  2  2  ...  2
2 3 2016
2 1
2017

1  22  23  ...  22016 : ª¬«22017  20 º¼» 1 Ÿ m2 n  3 31213 , m2 n  3 74 ˜13


2016

^ ` ^
Așadar, m2  1, 72 , 7 4 , adică m, n  1,31210 ; 7,634 ; 49,10 . `
G:734. Proiectul de matematică Ştiinţă şi Caracter din Şcoala noastră, are capitolele 1) Planificare, 2)
Parteneri, 3) Cursuri, 4) Corespondenţa la Gazeta Matematică, 5) Tabăra de Matematică şi Expediţia
1
Aura Pădurii, 6) Evaluare şi câte un număr de teme pentru fiecare. Capitolul 1 conţine din numărul
5
1 2 1
temelor, capitolul 2, din rest, capitolul 3, din noul rest, capitolul 4, din noul rest, capitolul 5,
8 7 5
1 1
din noul rest, capitolul 6, din noul rest. Ştiind că s-a mai parcurs o temă, aflaţi numărul total de
2 2
teme. Ion Stănescu, Buzău
Rezolvare : Se aplică metoda mersului invers.
MN = 1. KL = 2. IJ = 4 = GH = 5 = EF = 7 = CD = 8 =
AB = 10. Răspuns. 10 teme.

G:735. Dacă n  * , să se arate că 89n se poate scrie ca o sumă de trei pătrate perfecte distincte
nenule. Marin Chirciu, Pitești
Rezolvare: Cum 89 3  4  8 și 89 17  24  84 , atunci pentru n impar avem:
2 2 2 2 2 2 2

89k ˜ 89 (89k )2 ˜ 32  42  82 89 ˜ 3  89 ˜ 4  89 ˜ 8 .
2 2 2
89n 892 k 1 k k k

89 ˜17  89 ˜ 24  89 ˜ 84 , k 
2 2 2
Analog , pentru n par: 89n 892k  2 892k ˜ 892 k k k
.
1 1 3
G:736. Să se determine m, n  * care verifică relația:   1.
m n 2m n
Ionel Tudor, Viorica Dogaru, Giurgiu
Rezolvare:
Pentru n 1 sau m 1 membrul stâng este mai mare ca 1.
- PROBLEME REZOLVATE -

Fie m, n t 2 . Pentru n=2 rezultă (m  2)m 3 Ÿ m 3 . Pentru m²3, n² 2 Ÿ mn ²9 iar


1 1 3 1 1 1
  ¢   1 , ceea ce nu convine. Așadar, (m; n) ^ 3; 2 ` .
m n 2m n 3 2 6

G:737. Să se arate că nu există numere naturale x astfel încât x2017  x2015 x2016  x2014  2017 .
Marian Ciuperceanu, Craiova
Rezolvare: Presupunem prin absurd că relația dată este adevărată. Atunci,
x2017  x2016  x2015  x2014 2017 œ x2015 ˜ x( x  1)  x 2013 ˜ x ˜ ( x  1) 2017 œ
x( x  1) ˜ ( x2015  x2013 ) 2017, ceea ce este o contradicție deoarece membrul stâng este par iar membrul
drept este impar.

G:738. Determinați restul împărțirii numărului x 7n3 ˜ 2n2  7n2 ˜ 2n3  253 ˜14n la 2017.
Gheorghe Dârstaru, Buzău
Rezolvare:
x = 7n+3 2n+2 + 7n+2 2n+3 + 253 14n = 7n 73 2n 22 +7n 72 2n 23 + 253 14n
x = 14n( 73 22 + 72 23 + 253 ) = 14n ( 343 4 + 49 8+253) = 14n 2017
Restul împărțirii lui x la 2017 este 0 .

G:739. Fie numerele A 2  3  4 1  6  7  8  5  10  11  12  9  ...  238  239  240  237 şi


A 3B
B 1  2  3  4  5  6  7  8  9  ...  157 158  159 . Arătaţi că .
61 53
Sorina Văcărean, Cluj-Napoca
Rezolvare: Grupând termenii lui A câte 4, obţinem
60 ˜ 61 A
A 8  16  24  ...  480 8 1  2  3  ...  60 8 ˜ 240 ˜ 61 , de unde 240 . Grupând
2 61
termenii lui B câte 3, obţinem B 2  5  8  ...  158 . În sumă sunt 53 termeni, de unde

B
2  158 ˜ 53 80 ˜ 53 şi de aici 3B 240 , astfel că fracţiile date sunt echivalente.
2 53

ƒ Clasa a VI-a
G:740. Să se rezolve ecuația n  2017
n  2017
n  1765, n  .
Ionel Tudor, Călugăreni, Giurgiu
Rezolvare:
Numărul 2017 nu este soluție deoarece 00 nu are sens. Pentru 0 d n ¢ 2017 , fie m n  2017¢0, m .
p
Atunci, ecuația devine m m
m  252 . Fie m  p, p  * Ÿ ( p) 252  p (*).
1
Trecând la modul, 252  p ( p)  p ¢1 Ÿ 1¢252¢1 Ÿ 251¢ p¢253 Ÿ p 252 dar nu satisface
p
ecuația (*). Rezultă că ecuația nu are soluții pentru n ^1; 2;3;.....; 2017` .
Pentru n² 2017 și m n  2017  * ecuația mm m  252 se verifică doar pentru 4.

22017  3 ˜ 517
G:741. Să se arate că numărul este natural și să i se găsească ultima cifră .
17
Viorica Dogaru, Giurgiu
- PROBLEME REZOLVATE -

Rezolvare:
22017  3 ˜ 517 n
Fie n și k . Cum 22017  3 ˜ 517 15 ˜ 516  2 ˜ 216 17 ˜ 516  2 ˜ 516  2 ˜ 22016
17 17
M17  2 ª« 24  54 º» M17  2(17  1)504  2 ˜ 6254
504 4
M17  2(22016  517 )
¬ ¼
M17  2 M17  (1)504  2(17 ˜ 37  43)4 M17  2  2 ˜162 M17  2  2(167  1)2
M17  2  2(M17  1) M17 , așadar, n 17 Ÿ k  .

u 2 ˜ u(16504 )  3u(517 ) u 12  15 7 iar cum k


n
Ultima cifră a lui n este u(2 ˜16504  3 ˜ 517 )
17
rezultă n 17k Ÿ u(n) u(17k ) u 17 ˜ u(k ) 7 Ÿ u(k ) 1 .

G:742. Arătați că (a  1)n  a  1 se divide cu a, a  și că


n 1
(a  1)  2(a  1)  a  2a  3
2n 2
A  , a  *,, n  . Marin Chirciu, Pitești
a(a  1)n  a 2  3a
Rezolvare:
ª¬(a  1)n  a  1º¼ ˜ ª¬(a  1) n  a  3º¼ (a  1)n  a  1
(a  1)  a  1 Ma  1  a  1 a ;
n
A  .
a ª¬(a  1)n  a  3º¼ a

G:743. Fie a,b,c numere prime astfel încât 5a+7b, 7b+11c, și 11c+5a sunt direct proporționale cu 25,
56 respectiv 46. Arătați că a3  b3  c3 este divizibil cu 45.
Marian Ciuperceanu, Craiova
5a  7b 7b  11c 11c  5a 10a  14b  22c
Rezolvare: Din enunțul problemei rezultă kŸ
25 56 46 127
127k
5a  7b  11c . Din această relație și 5a  7b 25k , 7b  11c 56k , 5a  11c 46k rezultă
2
consecutiv c = 7, a = 3, b = 5. Înlocuind, obținem a3  b3  c3 495 45 ˜ 55 45 .

G:744. Să se calculeze minimul și maximul expresiei E = x+3y-2z, știind că x, y și z sunt numere


pozitive și avem 7x-2y+6z = 3 și 11x-6y+3z = 4.
Petre Rău, Galați
Rezolvare: Din condițiile date determinăm variabilele y și z în funcție de variabila x și găsim: y = ,

z= ; cum avem de a face cu trei variabile pozitive, rezultă că ≤ x ≤ . Expresia E devine: E = 6x-2,
deci Emin = 0 și Emax = 1.

G:745. Arătaţi că nu există numere naturale formate din trei cifre de 1 și restul cifrelor 0 care să poată
fi scrise ca sumă de două pătrate perfecte.
Neculai Stanciu, Buzău și Titu Zvonaru, Comănești
Rezolvare: Fie n numărul din enunţ; deoarece suma cifrelor sale este 3, n este divizibil cu 3 dar nu este
divizibil cu 9.
Presupunem că există numerele naturale a și b astfel încât n a 2  b 2 ; deoarece resturile pătratice
modulo 3 sunt 0 și 1, deducem că a b 0(mod 3) . Atunci a 2  b 2 (3k ) 2  (3 p) 2 9k 2  9 p 2 se
divide cu 9 , contradicţie cu faptul că n nu se divide cu 9!

x( x  1)( x  2)
G:746. Arătați că fracția este reductibilă pentru x, y, z numere naturale nenule, y
23 y  19 z
număr impar. Gheorghe Dârstaru, Buzău
- PROBLEME REZOLVATE -

Rezolvare:
este produsul a trei numere naturale consecutive, deci este divizibil cu 3
23 = (24 – 1)y = M24 – 1y = M24 – 1
y
19z = (18 + 1)z = M18 + 1z = M18 + 1
Numărătorul și numitorul multiplii de 3 rezultă că fracția este reductibilă
1 1 1
G:747. Să se rezolve în * ecuaţia:   1.
a b abc
Gheorghe Ghiţă, Buzău
1 1 1
Rezolvare: Pentru c = 0, ecuaţia devine:   1.
a b ab
1 1 1 1 1 1 5
Dacă a d b Ÿ 1   d   Ÿ 2a d 5 Ÿ a  {1,2}.
a b a  b a a 2a 2a
1 1
Pentru a = 1 avem: 1   1, ecuaţie imposibilă.
b 1 b
1 1 1
Pentru a = 2 ecuaţia devine:  . Din faptul că
b 2b 2
1 1 1 1 1 2 1 2
   Ÿ  Ÿ b  4 Ÿ b  {2,3}.
2 b 2b b b b 2 b
Se constată uşor că nici o valoare a lui b nu este soluţie. Deci, ecuaţia admite numai soluţii nenule.
2) Pentru c z 0 avem:
1 1
a) Dacă a 1Ÿ  0, ecuaţie imposibilă.
b 1 b  c
1 1 1
b) Dacă a 2Ÿ  ;
b 2bc 2
i) b=2, ecuaţie imposibilă;
1 1
ii) b 3Ÿ Ÿ c 1; soluţia este (2,3,1).
5c 6
1 1 1 1 1 1
iii) b ! 3 Ÿ    Ÿ ecuaţie imposibilă.
2 b 2bc 3 6 2
1 1 1 1 1 1 17
c) Dacă a, b t 3, Ÿ 1   d    1, fals Ÿ ecuaţie imposibilă.
a b a  b  c 3 3 7 21
Cum ecuaţia este simetrică în a şi b ecuaţia admite şi soluţia (3,2,1).
În concluzie, mulțimea soluţiilor S = {(2,3,1):(3,2,1)}.
G:748. Un unghi are 171 0 şi este împărţit în unghiuri cu măsurile 1 0, 2 0, ---, n 0, n  . În câte părţi
este împărţit unghiul dat ?
Ion Stănescu, Buzău
n(n  1)
Rezolvare : Cum 1  2  3  ...  n 171 Ÿ n(n  1) 34 18 ˜19 Ÿ n 180.
2
810
G:749. Arătaţi că 1.
286k  283k  280k  277k  274k  271k  ...  130k  127k
Sorina Văcărean, Cluj-Napoca
Rezolvare:
286k  283k  280k  277k  274k  271k  ...  130k 127k 30  30  30  ...  30 .
Şirul 1300, 1360, 1420, …, 2860 are 27 termeni, astfel că numitorul va fi 30 ˜ 27 810 , de unde rezultă că
fracţia dată este echiunitară.

ƒ Clasa a VII-a
- PROBLEME REZOLVATE -

G:750. Să se rezolve în u ecuațiile: a) x 2  y 2 2017; b) x 2  y 2 20172 ;


Ionel Tudor, Călugăreni, Giurgiu
Rezolvare:
a) Fie 1 d x¢ y . Atunci, y 2 ¢ x 2  y 2 2017 Ÿ y¢ 2017 . Cum 44¢ 2017 ¢ 45 Ÿ y d 44.
Atunci, y d 44
2
1936 Ÿ 2017  x d 1936 Ÿ x2 t 81 Ÿ x t 9 .
2 2

Soluția ( x; y) ^ 9; 44 ` este unică, unicitatea este dată de teorema lui Fermat.


Cum ecuația este simetrică atunci și perechea 44;9 este soluție.
b) Evident perechile (0; 2017) și 2017;0 sunt soluții. Fie x¢ y . Din identitatea lui Lagrange rezultă:
(a2  b2 )(c2  d 2 ) (ac  bd )2  (bc  ad )2 Ÿ
20172 (92  442 )(442  92 ) (9 ˜ 44  44 ˜ 9)2  (44 ˜ 44  9 ˜ 9)2 (2 ˜ 9 ˜ 44)2  (1936  81)2 7922  18552
Așadar, x; y ^ 0;2017 ,(2017;0),(792;1855),(1855,792)` .

G:751. Se consideră numărul A ab(a  2b  c)  bc(b  2c  a)  ca(c  2a  b) cu a, b, c nenule și distincte.


Arătați că numărul A se poate scrie ca suma a trei pătrate perfecte nenule.
Marin Chirciu, Pitești
Rezolvare:
A (10a  b)(a  2b  c)  (10b  c)(b  2c  a)  (10c  a)(c  2a  b). După desfacerea parantezelor obținem
A 8(a2  b2  c2 )  8(ab  bc  ca) 4(2a2  2b2  2c2  2ab  2ac  2bc) 4 ª¬(a  b)2  (b  c)2  (c  a)2 º¼ ,
evident fiind o sumă de trei pătrate perfecte nenule.

G:752. Determinați numerele x, y cu x t y și x  >1; f , știind că x2  y 2  x  3 y  xy  4 0.


Marian Ciuperceanu, Craiova
Rezolvare:

Relația dată este echivalentă cu y 2  4 y  4  ( x 2  xy  x  y) 0 œ ( y  2)2  ( x  1)( x  y) 0 .
Cum membrul stâng este pozitiv, rezultă x= y= 2.

G:753. Fie A ^1,9,17, 25,33,....` o mulţime cu n+1 elemente şi B ^8k 11 d k d n; n, k  *` .


a) Aflaţi numărul elementelor mulţimii A B;
b) Arătaţi că numărul elementelor mulţimii A×B nu este pătrat perfect ;

S 1  7  9  15  17  23  25  31  ....  8n  7  8n  1  8n  1
c) Calculaţi suma ;
x y
2 2
d) Arătaţi că oricare ar fi x şi y din AUB numărul este număr întreg.
16
Dana Badea, Ploiești
Rezolvare:
4k  4h 1 Ÿ 4 |1 fals
a) x  A œ x 8h  1;0 d h d n x  B =>
A B ‡ Ÿ card A B =0
Deci ,
card A u B n2 < n2  n < n  1
2
n2  n
b) şi .
c) S este suma elementelor mulţimii A plus suma elementelor mulţimii B
=> S
8n  2 n  1  8n  6 n  1 n  1 16n  8 n  1 8n  4
2 2 2
d) x, y  A ‰ B Ÿ x 8h  1 şi y 8k  1 Ÿ
- PROBLEME REZOLVATE -

x2  y 2 x  y x  y 8h  1  8k  1 8h  1  8k  1 16 4h  4k  1 h  k
4h  4k  1 h  k 
16 16 16 16

G:754. a) Scrieţi, ca diferenţă de 2 fracţii, o fracţie ce conţine termenii 1, a, a+1.


2017 § 1 1 · § 1 1 · 5 1 1 1
b) Cercetaţi egalitatea ¨  ¸  ..........  ¨  ¸    ..............  .
2018 © 2 2018 ¹ © 2017 2018 ¹ 6 4 5 2018
Ion Stănescu, Buzău
Rezolvare:
a)
b)
La stânga egalităţii date se reduc fracţiile opuse.

G:755. Determinaţi toate perechile x, y de numere întregi astfel încât


x 2 y 2  x 2  xy  x  y 2  y 0.
D.M. Bătineţu-Giurgiu, Bucureşti şi Neculai Stanciu, Buzău
Rezolvare:
x 2 y 2  x 2  xy  x  y 2  y 0 œ ( x 2 y 2  x 2  y 2  1)  ( xy  x  y  1) 0 œ
œ ( x 2  1)( y 2  1)  ( x  1)( y  1) 0 œ ( x  1)( y  1)>( x  1)( y  1)  1@ 0 .
Aşadar avem două familii de soluţii: ( x, y) (1, a) şi ( x, y) (b,1) , unde a şi b sunt numere întregi,
precum şi soluţiile care provin din ( x  1)( y  1)  1 0 .
­x  1 1 ­ x  1 1
Soluţiile ecuaţiei ( x  1)( y  1)  1 0 œ ( x  1)( y  1) 1 œ ® sau ®
¯y 1 1 ¯ y  1 1
sunt ( x, y) (2,2) respectiv ( x, y) (0,0) .
Prin urmare ecuaţia are soluţiile : ( x, y)  ^(1, a), (b,1), (0,0), (2,2)`, unde a  Z , b  Z .

G:756. Să se determine triunghiurile dreptunghice de laturi numere naturale şi raza cercului înscris
un număr prim.
Gheorghe Ghiţă, Buzău
Rezolvare : Cu notaţiile cunoscute, avem relaţia S = pr, unde a este ipotenuza triunghiului. Cum 2p = a +b +
c, şi pentru triunghiul dreptunghic 2S = bc, avem relaţia:
bc rb  rc bc
bc =r(a + b + c) œ a b  c
r r
şi cum a 2 b 2  c 2 , din cele două relaţii, deducem:
b2c 2 2b 2 c 2bc 2
 b 2
 c 2
   2bc b 2  c 2 œ bc  2rb  2rc  2r 2 0 œ (b  2r )(c  2r ) 2r 2 .
r2 r r
(*)
Din ultima relaţie rezultă că cel putin unul dintre numerele b sau c este număr par.
Fie b = 2n, n N * . Relaţia ultimă, devine:(n-r)(c-2r)=r2,
de unde rezultă că (n-r)/r2, şiatunci avem situaţiile:
1). n  r 1Ÿ n r 1Ÿb 2r  2, c r 2  2r, a r 2  2r  2;
2). n  r r Ÿn 2r Ÿb 4r, c 3r, a 5r;
3). n  r r 2 Ÿ n r 2  r Ÿb 2r (r 1).c 2r 1, a 2r 2  2r 1.
- PROBLEME REZOLVATE -

În concluzie, triunghiurile dreptunghice căutate sunt de laturi:


( r  2r  2;2r  2;r 2  2r ),(5r;4r;3r ),(2r 2  2r 1;2r 2  2r,2r 1), unde r este un număr prim.
2

G:757. Să se arate că 2k+22k+32k+…+92k se divide cu 470, pentru fiecare k natural impar.


Petre Rău, Galați
Rezolvare: Pentru un k fixat, fiecare termen se termină cu aceeași cifră cu care se termină și numărul 2 k.
Fiind zece termeni, înseamnă că numărul A se va termina cu 0, deci este divizibil cu 10.
Pe de altă parte, dacă grupăm câte doi termeni de la extremități, și știind că ak+bk este multiplu de (a+b) pentru
orice k natural impar, avem că numărul A se divide cu 2+92 = 12+82 = 22+72 = 32+62 = 42+52 = 94.
Așadar, numărul A se divide cu 470.

G:758. Triunghiul ∆ABC are m(∢A)=900, M∈AC, (BM este bisectoarea unghiului B şi N∈ (BC) astfel
încât [BN]≡[BA]. Arătaţi că :
a) N este simetricul lui A faţă de BM; b) NB·NC = AM·AC . c) MB·AC = AN·BC.
Dana Badea, Ploiești
Rezolvare:
a) Deoarece BN=BA⇒∆NBA isoscel şi cum (BM este bisectoarea ∢B⇒ BM⊥AN şi BM este mediatoarea lui
[AN], deci N este simetricul lui A faţă de BM.
b) ∆ABM ≡ ∆NBM (L.U.L.) ⇒[AM]≡[MN], m(∢MNB) = m(∢MAB)=900 ⇒ m(∢MNC)=900
CN MN
⇒∆CNM∼∆CAB (U.U.)⇒ = ⇒AC‧AM = CN‧AB de unde AM·AC =NB·NC .
AC AB
c) Din ∆ABM dreptunghic, AN⊥BM ⇒ ∢MAN ≡ ∢MBN ⇒∆ANC∼∆BMC (U.U.)⇒
AN AC
= ⇒AN‧BC=BM‧AC .
BM BC

G:759. În triunghiul ABC medianele AM și BN sunt perpendiculare (AM BN = ). Știind că GM


= 6 cm și BG = 8 cm, calculați perimetrul și aria triunghiului ABC.
Gheorghe Dârstaru, Buzău
Rezolvare:
Din GM = 6 cm avem AM = 18 cm și AG = 12 cm; Din BG = 8 cm avem BN = 12 cm și GN = 4 cm
Teorema lui Pitagora în triunghiul BGM rezultă BM = 10 cm și BC = 20 cm
Teorema lui Pitagora în triunghiul BGA rezultă BA = 4 cm
Teorema lui Pitagora în triunghiul AGN rezultă AN = 4 cm și AC =8 cm
BN ˜ AG 12 ˜12
PABC 8 10  4 13  20 ; AABN 72cm2 ACBN (proprietatea medianei)
2 2
AABC = 144 cm2 .

ƒ Clasa a VIII-a
G:760. Să se determine mulţimile de valori pe care le poate lua numerele reale a, b, c dacă
2a 2  b 2  c 2  2ac  6a  6b  2c 2.
Constantin Rusu, Râmnicu Sărat
Rezolvare: Relaţia din enunţ se scrie (a  2)  (b  3)  (a  c  1)
2 2 2
16 . Deci,
(a  2) 2 d 16 œ a  2 d 4 œ 4 d a  2 d 4 œ a  [2,6] , (1).
(b  3) 2 d 16 œ b  3 d 4 œ 4 d b  3 d 4 œ b  [1,7] , (2).
(a  c  1) 2 d 16 œ a  c  1 d 4 œ 3 d a  c d 5 , (3).
- PROBLEME REZOLVATE -

Din (1) rezultă  6 d a d 2 , (4). Adunând (3) cu (4) obţinem c  [9,7] , (5).
Deci, a  [2,6], b  [1,7], c  [9,7] .

G:761. Să se determine toate tripletele( p, q, r) de numere prime, soluții ale ecuației:


p  50q  450r 2017.
Ionel Tudor, Călugăreni, Giurgiu
Rezolvare:
Cum 50q  450r este număr par, atunci p trebuie să fie prim impar cu ulima cifră 7, deci p este de forma
10s+7 cu s natural.
Ecuația devine: 10s  7  50q  450r 2017 Ÿ s  5q  45r 201 Ÿ s  1  5q  45r 200 . De aici,
rezultă că s-1 trebuie să fie divizibil cu 5, deci s-1= 5k iar s = 5k+1.
Mai departe se obține ecuația: 5k  5q  45r 200 : 5 Ÿ k  q  9r 40 , k  , q,r prime.
Dacă r t 5 este prim, atunci 40 k  q  9r²9r t 45 , fals. Așadar, r  ^2;3` .
Fie r =2. Din k+q=22 și q prim rezultă q ^2;3;5;7;11;13;17;19` .
Se obțin soluții doar pentru q=3 și q=13, q= 19.
Dacă q=3 rezultă k= 19 iar s=96. Atunci p = 967 e prim. ( p1; q1; r1 ) 967;3; 2 .
Dacă q= 13 rezultă k=9 iar s= 46. Atunci, p= 467 e prim. ( p2 ; q2 ; r2 ) 467;13; 2 .
Dacă q = 19 rezultă k=3 iar s= 16. Atunci, p= 167 e prim. ( p3 ; q3 ; r3 ) 167;19; 2 .
Fie r = 3.
Din ecuația k  q  9r 40 Ÿ k  q 13, k  iar cum q este prim rezultă q ^2;3;5;7;11;13` .
Dacă q = 7 rezultă k= 6 iar s = 31. Atunci p = 317 e număr prim. ( p4 ; q4 ; r4 ) 317;7;3 .
Dacă q = 13 rezultă k= 0 iar s = 1. Atunci p = 17 e număr prim. ( p5 ; q5 ; r5 ) 17;13;3 .
8 x2  7 .
3 3 3
G:762. Să se determine x  știind că: x 2  3x  4  x 2  3x  10
Iuliana Trașcă, Olt
Rezolvare: Fie a x  3x  4, b
2
x  3x  10 Ÿ a  b 2 x  7
2 2

Relația din enunț devine a3  b3 a  b


3

sau a  b a 2  ab  b2 a  b 3
œ a  b ˜ ab 0
Avem 2 situații :
i) a  b ^
0 Ÿ x 2  7 0 Ÿ x   7, 7 ; `
ii) a ˜ b 0 Ÿ x 2  3x  4 x 2  3x  10 0 , x  4 x 1 x  5 x  2 0 œ x  ^4,  2, 1, 5`

^
Concluzie x  4,  7,  2, 1, 7, 5 . `
2b 2  6b  3
G:763. Să se determine valoarea minimă a raportului A , unde numerele naturale a,
2a  5
b sunt pare, consecutive, crescătoare în această ordine.
Marian Ciuperceanu, Craiova
Rezolvare:
2(a  2) 2  6(a  2)  3 2a 2  14a  23
Înlocuind b= a+2 în enunț, obținem: A . Dacă A ia valoarea
2a  5 2a  5
minimă, atunci și dublul său va lua valoarea minimă
4a 2  28a  46 (2a  5) ˜ (2a  9)  1 1 1
2A 2a  9  2a  5   4 t 2  4 6 . Așadar,
2a  5 2a  5 2a  5 2a  5
1
2 A t 6 Ÿ A t 3 , minimul lui A fiind 3. S-a folosit relația x  t 2, x   *.
x
- PROBLEME REZOLVATE -

G:764 Dacă a, b, c ! 0 si a 2  b2  c 2 1, arătați că are loc inegalitatea:


1 1 1 1
  d . Florin Stănescu, Găești, Dâmbovița
a  b b  c c  a 2abc
Rezolvare:
1 1 1  1
2
4 1 1 4 1 1 4
Putem scrie:  t și analoagele:  t ,  t ,
a b ab ab b c bc c a ca
1 1 1 § 1 1 1 ·
de unde   t 2¨   ¸. În continuare vom arăta că :
a b c ©ab bc ca¹
1 1 1 a b c
  d   œ ab  bc  ca d a 2  b2  c 2 œ
a b c bc ca ab
0 d a  b  b  c  c  a ,
2 2 2

Astfel,
a b c § 1 1 1 · a 2  b2  c 2 1 1 1
  t 2¨   ¸ Ÿ t   Ÿ
bc ca ab ©ab bc ca¹ 2abc ab bc ca
1 1 1 1
  d .
a  b b  c c  a 2abc
k k
§ 1· § 1· k 1
G:765. Fie a și b două numere reale strict pozitive, cu ab ≤ 1. Să se arate că ¨1  ¸  ¨1  ¸ t 2 .
© a¹ © b¹
Petre Rău, Galați
k k k k
§ 1· § 1· § 1· § 1·
Rezolvare: Fie E (a; b) ¨1  ¸  ¨1  ¸ . Atunci E (a; b) t 2 ¨1  ¸ ˜ ¨1  ¸
© a¹ © b¹ © a¹ © b¹
k k 2k k
§ ab 1 · § 2 1 · § 1 · § 1 ·
2 ¨1   ¸ t 2 ¨1   ¸ t 2 ¨1  ¸ 2 ¨1  ¸ . Dar ≤ 1, deci
© ab ab ¹ © ab ab ¹ © ab ¹ © ab ¹
1+ ≥ 2, de unde rezultă că E(a,b) ≥ 2k+1.
n


¦ f (i) 1
G:766. Funcţia f : N o Q , satisface condiţia: f (n)

i 1
2
, n  N . Dacă f (1) ,
n 2
2017
determinaţi ¦ f (i) .
i 1
D.M. Bătineţu-Giurgiu, Bucureşti şi Neculai Stanciu, Buzău

Rezolvare: Scădem relaţiile: f (1)  f (2)  ...  f (n) n 2 f (n) ;


f (1)  f (2)  ...  f (n  1) (n  1) 2 f (n  1) , şi obţinem
(n  1) 2 n 1
f ( n) f (n  1) f (n  1) . Prin telescoparea formulei precedente obţinem:
n 1
2
n 1
(n  1)! 2 f (1) 1 1 1 1
f ( n) 2 ˜ ˜ f (1)  .Deci, f (n)  .
(n  1)! n(n  1) n n  1 n n 1
1 1 2017
Atunci, f (1)  f (2)  ...  f (2017) =  .
1 2018 2018
G:767. Rezolvaţi în mulţimea numerelor naturale ecuaţia: ª x 2 3 º  ª x 12 º  ª 299 º 2017
¬ ¼ ¬ ¼ ¬ ¼ .
Dana Badea, Ploiești
Rezolvare: Din ª 299 º ª 2 º ª º
¬ ¼ 17 Ÿ ¬ x 3 ¼  ¬ x 12 ¼ 2000 .
- PROBLEME REZOLVATE -

Se deduce că orice soluţie verifică x d 33 . Se arată că x 33 este soluţie


Se arată că numerele naturale x  33 nu sunt soluţii.

G:768. Rezolvați în mulțimea numerelor întregi ecuația: 18x2 – 12x = 3xy – y .


Gheorghe Dârstaru, Buzău
Rezolvare:
y(3x – 1) = 18x2 – 12x y ,

2(3x – 1 ) – 3x– 1 D2, 3x – 1 cu x = 0 și y= 0 sau x = 1și y = 3.

1 1 1 2n
G:769. Să se arate că :  ... t , nN * .
1 2 n n 1
Gheorghe Ghiţă, Buzău
1 1 4 1 1 4
Rezolvare: În inegalitatea:  t , a, b ! 0,  t œ (a  b) 2 t 4ab œ (a  b) 2 t 0, a, b ! 0,
a b a b a b ab
1 1 4
considerăm a k ,b k 1, kN * , şi atunci aceasta devine:  ! 4( k 1  k ) .
k k 1 k  k 1
______
Pentru k 1.n 1, obţinem inegalităţile:
1 1
 ! 4( 2  1),
1 2
1 1
 ! 4( 3  2 ),
2 3
. 1 1 4 4( n 1)
. şi cu ajutorul inegalităţii:  ! ,
. n 1 n 1 n 1
1 1
 ! 4( n  n 1),
n 1 n
prin adunarea lor, obţinem inegalitatea:
§ 1 1 1 · 4( n 1) § 1 · 4n 1 1 1 2n
2¨¨  ... ¸¸ ! 4( n 1)  4( n 1)¨1 ¸ . œ   ... !
© 1 2 n¹ n 1 © n 1 ¹
n 1 1 2 n n 1
Cum inegalitatea are loc şi pentru n = 1, devenind egalitate, rezultă că inegalitatea propusă este
adevărată.
G:770. Se consideră ABC echilateral. În exteriorul triunghiului se construiesc pătratele ABDE ,
BCGF şi ACHI .
a) Arătaţi că aria suprafeţei nonagonului concav AEDBFGCHI este mai mică decât aria suprafeţei
pătratului AKLM , unde K este mijlocul laturii > FG @ .
b) Arătaţi că aria suprafeţei hexagonului convex EDFGHI este de patru ori mai mare decât aria
suprafeţei NFG , unde N este mijlocul laturii > EI @ .
Sorina Văcărean, Cluj-Napoca
Rezolvare: Fie a lungimea laturii ABC .

a) AAEDBFGCHI A ABC  3 ˜ AABDE


a2 3
 3a 2

a 2 12  3 . Fie P mijlocul laturii > BC @ .
4 4

Punctele A , P şi K sunt coliniare, astfel că AK AP  PK


a 3
a

a 2 3 .
2 2
.
2

2
a2 2  3 a2 7  4 3
AAKLM AK
4 4
- PROBLEME REZOLVATE -

AAEDBFGCHI  AAKLM œ

a 2 12  3  a 7  4 3 œ 12 
2

3  7  4 3 œ 5  3 3 œ 25  27 A .
4 4

b) AEDFGHI AAEDBFGCHI  3 ˜ A BDF . Cum A BDF A ABC Ÿ AEDFGHI



a 2 12  3  3˜ a 2
3
4 4

a a 2 3

a 2 3  3 . Punctele N , A şi K sunt coliniare, astfel că NK NA  AK
2

2

a 3 3 . A FG ˜ NK
a 3 3 ˜1
a2 3  3 . A
2
NFG
2

2 2 4
EDFGHI 4 ˜ A NFG œ a 2 3  3

a2 3  3 A .
4
G:771. Cubul ABCDAI B I C I DI are AB = 1, O = centrul cubului, O 1 =centrul feţei ABCD,
O 2 =centrul feţei AI B I C I D I , M = mijlocul lui ª¬ AAI º¼ , N = mijlocul lui CC I .

a) Găsiţi un triunghi cu perimetrul


1
2
1 2  3 .
2
b) Identificaţi un romb cu aria .
2
Ion Stănescu, Buzău
2 1 3
Rezolvare: Triunghiul AI O2O are AI O2 , O2O , AI O . Perimetrul este
2 2 2
P
1
2

1 2  3 .
2
b) Rombul O2 NO1M are aria .
2

ƒ Clasa a IX-a
L:553. Să se demonstreze că 1  3  5  .....  2n  1  2n  1 d (n  1) n  1 .
Marian Ciuperceanu, Craiova
Rezolvare: Se folosește inegalitatea mediilor dintre cea aritmetică și cea pătratică.
1  2n  1
1  2n  1 d 2 2 n 1 ,
2
3  2n  1
3  2n  1 d 2 2 n 1 ,
2
…………………
Prin însumarea celor n+1 termeni din stânga, sau a celor ( n+1):2 rânduri rezultă cerința.
an
L:554. Considerăm şirul a n nt1 definit prin a1 1 şi a n 1 , n t 1 . Să se determine a n şi să se
1  na n
- PROBLEME REZOLVATE -

n
k
calculeze ¦a
k 1
. D.M. Bătineţu-Giurgiu, Bucureşti şi Neculai Stanciu, Buzău
k

1 1  na n 1
Rezolvare: Se arată prin inducţie că an ! 0, n . Apoi,  n, n .
a n 1 an an
1 1 § 1 1 · n1
n 1
n(n  1) 1 n(n  1)
Astfel că, ¦
a n a1
¨
¨a
k 1 © k 1
 ¸¸ ¦ k
ak ¹ k 1 2
. Deci,
an
1
2
. Aşa că,

1 2012 ˜ 2013 1 1 k (k  1)
1 2025079 , şi urmează a 2013 . Din 1 avem:
a 2013 2 2025079 ak 2
n
k n
ª k (k  1) º n § k 2 (k  1) · n
1 n 3 1 n 2
¦ = ¦ k ˜ «
¬
1  = ¦
2 »¼ k 1 ¨©
¨ k  ¸¸ ¦k  ¦k  2 ¦ k =
k 1 ak k 1 2 ¹ k 1 2k1 k 1

1ª n 2 (n  1) 2 n(n  1)(2n  1) º
« n ( n  1)   » .
2¬ 4 6 ¼

L:555. Sӑ se scrie aproximӑrile zecimale prin lipsӑ și prin adaos cu eroare de 102 pentru numӑrul
2 3 4 18 19 20
n    ......    .
3 ˜ 5 5 ˜ 8 8 ˜12 155 ˜173 173 ˜192 192 ˜ 212
Gheorghe Dârstaru, Buzău
Rezolvare: n= - + - +…+ - + - ,n= - ,n= 0,32.

L:556. Dacă a, b, c t 0 și a3  b3  c3 192 , să se arate că 9  a 2  9  b2  9  c2 d 15.


Adrian Stan, Buzău
Rezolvare:


9  a 2  9  b2  9  c2 d 12  11  12 ˜ 27  a 2  b2  c2 d 3 ˜ 75 225 Ÿ
2
9  a2  9  b2  9  c2 d 15.

a 3  b3  c 3 a 2  b2  c 2 192 a 2  b2  c 2 a 2  b2  c 2
S-a folosit faptul că 3 t œ3 t Ÿ4t
3 3 3 3 3
Ÿ a2  b2  c2 d 48 .
n
ai n 2
L:557. Dacă ai  , i  ^1,2,..., n`, atunci să se demonstreze că ¦a d .
i 1
2
i 2 4
Constantin Rusu, Râmnicu Sărat
ai ai D
2˜ ai
tg
a 2 2 2 2 2
Rezolvare: 1. Avem 2 i ˜ 2 ˜
ai  2 ª§ ai · 2 º 4 § a ·
2

2«¨ ¸  1» 1 ¨ i ¸
«¬© 2 ¹ »¼ © 2¹
ai D D
tg 2tg
2 2 2 2 2 2
˜ sin D d , de unde sumând pentru i 1, n obţinem concluzia.
4 D 4 4
1  tg 2
2
Soluţia 2. Folosim metoda inducţiei complete.
a1 2
Pentru n d
1, avem œ 2 (a1  2 ) 2 t 0 , adevărat.
a 2 4 2
1
Presupunem inegalitatea adevărată pentru orice n număr natural şi o demonstrăm pentru n  1 .
- PROBLEME REZOLVATE -

n
ai n 2 a 2
Deci, ¦a
i 1
2
2
d
4
şi la fel ca în cazul pentru n 1avem 2 n d
an  2 4
, ceea ce implică
i
n 1
ai n 2 a n 2 2 (n  1) 2
i 1
¦a2
d 2
4
 2 n 1 d
a n 1  2 4

4 4
.
i
x x
Soluţia 3. Considerăm funcţia f : o , f ( x) . Notăm y œ x 2 y  x  2 y 0 şi
x 2
2
x 2
2

ª 2 2º 2
deoarece x  R rezultă că ' x 1  8 y 2 t 0 œ y  « , » . Deci, f ( x) d este adevărată pentru
¬ 4 4 ¼ 4
ai  R , i  ^1,2,..., n` care prin sumare conduce la concluzia dorită.

L:558. Demonstrați că (3  4sin 2 90 )(3  4sin 2 270 ) ctg90.


Marin Chirciu, Pitești
Rezolvare:
(3sin 90  4sin 3 90 )(3sin 270  4sin 3 270 ) sin 3 ˜ 90 sin 3 ˜ 270
(3  4sin 2 90 )(3  4sin 2 270 ) ˜
sin 90 ˜ sin 270 sin 90 sin 270
sin 810 cos 90
ctg 90 .
sin 90 sin 90

(¦ a 2  ¦ ab) t ¦ a  b .
16 2
L:559. Arătaţi că dacă a, b, c ! 0 , atunci
3
Neculai Stanciu, Buzău și Titu Zvonaru, Comănești
Rezolvare: Fără a restrânge inegalitatea, putem presupune că a d b d c ; fie x, y t 0 astfel încât
b a  x, c a  x  y . Deoarece

¦ a  ¦ ab
2 1
2

(a  b) 2  (b  c) 2  (c  a) 2 1 2
2

x  y 2  ( x  y) 2 x 2  xy  y 2 ,
inegalitatea de demonstrat este echivalentă cu
16( x 2  xy  y 2 ) t 3( x  y  x  y) 2 œ 4( x  y) 2 t 0 , evident adevărată.

L:560. Să se rezolve ecuația > x @  > 2 x @  >3x @ 6x .


Ionel Tudor, Călugăreni, Giurgiu
Rezolvare:
Se observă că orice număr întreg n verifică ecuația dată. Arătăm că x  \^ ` nu verifică ecuația dată.
Conform proprietății n > x @ d > nx @ d n > x @  n  1, x  , n  *, înlocuind pe n cu 2 și cu 3 și adunând
relațiile, se obține: 5> x@ d > 2 x@  >3x @ d 5 > x@  3, Ÿ 6 > x @ d > x @  > 2 x @  >3x @ d 6 > x @  3, x  .
1 1 1
Dacă x este soluție a ecuației atunci 6 > x @ d 6 x d 6 > x @  3 Ÿ > x @ d x d > x @  Ÿ x  > x @ d , ^x` d .
2 2 2
^ x`  ª«0;

Dacă D Ÿ x n D, n  . Dacă D 0 Ÿ x n  e soluție.
¬ 2 »¼
§ 1·
Dacă D  ¨ 0; ¸ Ÿ > x @ > n  D @ n  >D @ n. Atunci, > 2 x @ > 2n  2D @ 2n  > 2D @ 2n,
© 2¹
>3x@ >3n  3D @ 3n  >3D @ . În acest caz ecuația devine n  2n  3n  >3D @ 6(n  D ) Ÿ >3D @ 6D.
­ 1
0¢ 6D , daca 0¢D ¢
Dar 0¢3D ¢ 3 Ÿ 3D
> @
°
°
®
3 . De aici rezultă că >3D @ 0¢6D și
2 1
°1, daca d D d 1
°̄ 3 2
- PROBLEME REZOLVATE -

1 ª1 1 º
>3D @ 1 6D Ÿ D  ; , deci nu avem soluții în acest caz. Singurele soluții sunt S .
6 «¬ 3 2 »¼
L:561. Să se rezolve ecuaţia: >x  7@  >x  11@  ...  >x  4n  3@ 2 ˜ n 2  6n, n  N * , unde >x @ este
partea întreagă a lui x.
Constantin Ciobîcă , Fălticeni
Rezolvare:
> x  7@  > x  11@  ...  > x  4n  3@ > x@  7  > x@  11  ...  > x@  4n  3 n ˜ >x@  7  11  ...  4n  3
7  11  ...  4n  3 4  3  4 ˜ 2  3  ...  4n  3 4  4 ˜ 2  ...  4n  3 3  
...

de n ori

n n  1
4 ˜ 1  2  ...  n  3n 4˜  3n 2 ˜ n 2  2n  3n 2 ˜ n 2  5 ˜ n
2
n ˜ >x@  2n 2  5n 2n  6n . Rezultă n ˜ >x@ n : n Ÿ >x@ 1 Ÿ 1 d x  2 Ÿ x  >1,2 .
2

L:562. a) Daca a, b, c t 0, astfel incat a  b  c 3, arătați că:



abc 1  a 2
1  b 1  c d 8.
2 2

n n
b) Daca x1 , x2 ,..., xn t 0 , astfel incat ¦
k 1
ak ¦ a , aratați că are loc inegalitatea:
k 1
k

1  a12  1  a22  ...  1  an2 d n ˜ 2.


Florin Stănescu, Găești, Dâmbovița
Rezolvare:
a)Cum a  b  c 3, din inegalitatea mediilor, putem scrie:
3 3
1 a 1 b 1 c · §33·
1  a 1  b 1  c d §¨ ¸ ¨ ¸ 8.
© 3 ¹ © 3 ¹
Mai departe, tot cu ajutorul inegalității mediilor, avem:


abc 1  a 2 1  b2 1  c 2 1
8

˜ 2a 1  a 2 ˜ 2b ˜ 1  b 2 ˜ 2c 1  c 2 d
2 2 2
1 § 2a  1  a 2 · § 2b  1  b 2 · § 2c  1  c 2 · 1 1
ª¬ 1  a 1  b 1  c º¼ d
4
˜¨ ¸ ¨ ¸ ˜¨ ¸ ˜ 84 23 8.
8 © 2 ¹ © 2 ¹ © 2 ¹ 8 ˜ 64 8 ˜ 64
b) Daca x t 0, cu ajutorul inegalității C-B-S, putem scrie:

d 1  1 ª« 2 x  1  x 2 º»
2 2
2 1  x ,
2 2
1 ˜ 2 x  1 ˜ 1  x2 2 1  2 x  x2
¬ ¼
x2  1
2 x  1 ˜ 1  x 2 d 2 1  x 2 1  x Ÿ x 
2
de unde , d x 1 Ÿ
2
x2  1
d 1 x  x .
2
Acum, luând pe rând în locul lui x a1 , a2 ,...an obținem:
ak2  1
d 1  ak  ak , k 1, n, iar prin însumare,
2
n
ak2  1 § n n
· n

¦
k 1 2
d n  ¨ ¦ ak  ¦ ak ¸
© k1 k 1 ¹
n Ÿ ¦ ak2  1 d n ˜ 2.
k 1
- PROBLEME REZOLVATE -

L:563. Fie funcţia f : >1, f o R, f x x  2 ˜ x  1  x  2 ˜ x  1 . Arătaţi că f este funcţie


constantă pentru x  >1,2 .
Constantin Ciobîcă , Fălticeni
Rezolvare:
f x x 1 2 ˜ x 1 1  x 1  2 ˜ x 1 1
2 2
x 1  2 ˜ x 1 1  x 1  2 ˜ x 1 1
x 1 1  2
x 1 1 2
x 1 1  x 1 1
x  >1, 2 Ÿ 1 d x  2 Ÿ 0 d x  1  1 Ÿ 0 d x  1  1
Ÿ 1 d x  1  1  0 Ÿ x 1 1 1  x 1 Ÿ 1 d x 1  1  2 Ÿ x 1  1 x 1  1
x  >1,2 Ÿ f x x 1 11  x 1 2 Ÿ f este funcţie constantă.

L:564. Determinaţi valoarea minimă a expresiei


E ( x; y) x 2  y 2  x 2  ( y  3)2  ( x  3)2  y 2  ( x  3)2  ( y  3) 2 . Laura Tănase, Buzău
Rezolvare: Folosind inegalitatea lui Minkovski obţinem

Se obţine egalitate pentru

Se obţine egalitate pentru

Adunând cele două inegalităţi, se obţine .


Egalitatea este adevărată pentru ,deci minimul expresiei este .

L:565. Să se arate că pentru orice numere reale x, y, z și orice k natural impar, expresia:
F(x,y,z) = (x+y+z)k - (-x+y+z)k - (x-y-z)k - (x+y-z)k se divide cu 8xyz.
Petre Rău, Galați
Rezolvare: Facem transformarea: -x+y+z=2a, x-y+z=2b, x+y-z=2c de unde avem x= , y= , z= și
x+y+z=2(a+b+c), 8xyz=(a+b)(a+c)(b+c), și problema devine:
F(a,b,c) = 2k[(a+b+c)k - ak - bk - ck] trebuie să se dividă cu produsul (a+b)(a+c)(b+c). Folosind Bézout, dacă: a
= -b avem că F(-b,b,c) = 0 deci F se divide cu a+b. La fel găsim și că F(-c,b,c) = 0, deci F se divide cu a+c și
F(a,-c,c) = 0, deci F se divide cu b+c. În final avem că F(a,b,c) se divide cu (a+b)(a+c)(b+c).

L:566. Fie ABC un triunghi oarecare de laturi a , b , c . Notăm cu la , ha , bisectoarea , respectiv


la2 lb2 lc2 a 2  b2  c 2
înălţimea corespunzătoare laturii a=BC , etc . Demonstraţi că max{ 2 , 2 , 2 } t .
ha hb hc ab  bc  ca
Vasile Jiglău , Arad
Rezolvare: Putem presupune că a d b d c . Atunci m( A) d m( B) d m(C ) (1)
la lb lc lb
Să observăm ca în aceste condiții are loc : max{ , , } (2)
ha hb hc hb
ha B C
Într-adevăr , este cunoscut faptul că într-un triunghi arbitrar are loc cos ;
la 2
- PROBLEME REZOLVATE -

lb lc BA CA
De aici rezultă că t œ cos t cos œ m(C ) t m( B) - adevarată , conform (1) .
hb hc 2 2
lb l
Asemănător t a În condițiile de mai sus , avem de demonstrat că :
hb ha
lb2 a 2  b 2  c 2 b 2 4acp( p  b) a 2  b 2  c 2
t œ ˜ t œ
hb2 ab  bc  ca 4S 2 (a  c) 2 ab  bc  ca
(ab  bc  ca)ab2c t (a 2  b2  c 2 )(a  c)2 ( p  a)( p  c)
4(ab  bc  ca)ab2c t (a 2  b2  c 2 )(a  c)2 (b2  (c  a)2 )
œ (c  a)2 (c  a)2 (a2  b2  c2 )  4ab2c(ab  bc  ca) t b2 (a  c)2 (a 2  b2  c 2 ) ( 3)
Din (1) rezultă că există m, n numere reale pozitive astfel încât b=a+m , c=a+m+n .
Înlocuind , prima parte a lui (3 ) devine , după efectuarea operațiilor :
12a 6  52a5 m  20a5 n  100a 4 m2  92a 4 mn  20a 4 n2  100a3m3  160a3m2 n  88a3mn2  20a3 n3  55a 2 m4
136a 2 m3n  134a 2 m2 n 2  68a 2 mn3  15a 2 n4  16am5  58am4 n  88am3n2  72am2 n3  32amn4  6an5 
2m6  10m5n  21m4 n2  24m3n3  16m2n4  6mn5  n6 iar a doua :
12a 6  52a 5m  20a 5n  95a 4m 2  78a 4mn  15a 4n 2  94a 3m3  122a 3m 2n  50a 3mn 2  6a 3n 3
53a 2m 4  96a 2m3n  62a 2m 2n 2  16a 2mn 3  a 2n 4  16am5  38am 4n  34am3n 2  14am 2n 3
2amn 4  2m6  6m5n  7m 4n 2  4m 3n 3  m 2n 4

Din compararea coeficienților polinoamelor de mai sus rezultă imediat că prima parte a lui (5) e mai
mare sau egală cu cea de-a doua , deci (3) e adevărată , astfel că și inegalitatea din enunț e adevărată .
Egalitatea se realizează pentru cazul în care m=n=0 , deci pentru cazul triunghiului echilateral .

Comentariu : problema propusă clarifică raportul dintre inegalitățile cunoscute


R lb2 R a 2  b2  c 2
t 2 și t , care apare în GMB nr 3/1990 .
2r hb 2r ab  bc  ca

ƒ Clasa a X-a
x 4
L:567. Să se rezolve ecuația 11  log 2 4 x . Marin Chirciu, Pitești
x  48 4
x 1
Rezolvare:
Condițiile de existență impun x²1. Folosind inegalitatea mediilor se obține:
x4  48 x4  16  16  16 t 4 4 x4 ˜16 ˜16 ˜16 4 ˜ x ˜ 2 ˜ 2 ˜ 2 32 x Ÿ
x 1 x 1 x
d Ÿ log 2 4 d log 2 5 Ÿ 11  log 2 4 d 6 . (1)
x  48 32
4
x  48 32 x  48
4 4 1 1 1 1 1 1 1 1
x 4 x 1 4  1 t 4      1 t 5 5 t 4 ˜ ˜ ˜ ˜  1 6 (2).
x 1 x 1 t t t t t t t t
Din (1) și (2) rezultă că avem egalitate în enunț dacă și numai dacă x= 2; 2 este soluția unică.

L:568. Rezolvaţi în ecuaţia:
(14  11 )(61  50 )(60  30 x )  (103 x  100 x )(14 x  3 x )(62 x  31x )
x x x x x
2013 x .
D.M. Bătineţu-Giurgiu, Bucureşti şi Neculai Stanciu, Buzău
- PROBLEME REZOLVATE -

Rezolvare: Observăm că x 1 este soluţia ecuaţiei.Vom arăta ca ecuaţia are soluţie unică.
Deoarece toate cantităţile sunt pozitive pentru x ! 0 , împărţim prin 2013 x (3 ˜ 11 ˜ 61) x şi obţinem ecuaţia
§ § 14 · x x ·§ x § 50 · x · x x § § 103 · x § 100 · x ·§ § 14 · x x · x x
echivalentă: ¨ ¨ ¸  1 ¸¨1  ¨ ¸ ¸ 20  1  ¨ ¨
¨ © 11 ¹ ¸¨ ¸
 ¸¨
¨ © 61 ¸¹ ¨© 61 ¸¹ ¸¨ ¨© 3 ¸¹
1 ¸ 2 1
¸
1.
© ¹© © 61 ¹ ¹ © ¹© ¹
Deoarece, cei doi termeni din membrul stâng sunt funcţii strict crescătoare (dacă a ! b t 1 atunci funcţia
a  b x este strict crescătoare şi produsul unor funcţii pozitive strict crescătoare este de asemeni strict
x

crescătoare pe >0, f şi suma unor funcţii pozitive strict crescătoare este de asemeni strict crescătoare pe
>0, f , rezultă că soluţia x 1 este unică.
L:569. Rezolvați în R ecuațiile:
x7 x 8

a) 10 ˜ 4log2017 x  25log2017 x 29 ˜10log2017 x ; b)
x 8

x7
17
4
.
Gheorghe Dârstaru, Buzău
Rezolvare:
a) Se impune x² 0 , și notația log 2017 x t . Ecuația dată devine: 10 - 29 + 10 = 0, împӑrțim
prin și notӑm =y 0 de unde t1 = 1, t2= - 1 și x1 = 2017, x2 =

b) x ( - ; -8) (7; + ), Notӑm = y 0 de unde y1 = 4, y2 = și x1 = - 9 și x2 = 8.

L:570. Fie x log3 5  log5 7  log7 3. Să se calculeze [x]. Petre Rău, Galați
Rezolvare: Aplicând inegalitatea mediilor, rezultă că x = log35+log57+log73 = + + ≥

3 ≥ 3. Dar log35< log3 = , log57 < log5 = , log73 < log7 = Așadar, x < + +

= < 4. Cum 3≤x<4, rezultă [x] = 3.

4x 25x 10 x
L:571. Să se rezolve în  ecuația  . Adrian Stan, Buzău
log 2 ( x  1) log5 ( x  1) lg( x  1)
2x x
§2· § 2·
Rezolvare: Ecuația dată este echivalentă cu 4 x lg 2  25x lg 5 10 x : 25x Ÿ ¨ ¸ lg 2  ¨ ¸  lg 5 0.
©5¹ ©5¹
x
§2·
Notăm cu ¨ ¸ t ² 0 , rezultă ecuația lg 2 ˜ t 2  t  lg 5 0 cu soluțiile t1 1 Ÿ x1 0;
©5¹
lg 5 § lg 5 ·
și t2 Ÿ x2 log 2 ¨ ¸.
lg 2 5 © lg 2 ¹

L:572. Fie a, b, c ! 0 astfel astfel incat a  b  c 3. Aratati ca are loc inegalitatea:


a
2
2
 b2  c 2 ab  bc  ca
 n˜ t 3  n, unde 0 d n d 2.
a bb cc a
2 2 2
a 2  b2  c 2
Florin Stănescu, Găești, Dâmbovița
Rezolvare:
Pentru început vom arăta că dacă a, b, c ! 0 astfel astfel încat a  b  c 3, atunci
a  b  c t a b  b c  c a. Astfel, 3 a  b  c
2 2 2 2 2 2 2 2 2
a 3  c  b 3  a  c2 3  b +
2 2

ab2  bc 2  ca 2 a 2 a  b  b2 b  c  c 2 c  a  ab2  bc2  ca2 a3 


- PROBLEME REZOLVATE -

ab2  bc 2  ca 2 a 2 a  b  b2 b  c  c 2 c  a  ab2  bc2  ca2 a3 


b3  c3  a 2 b  ab2  b2 c  bc 2  c 2 a  ca 2 a 3

 a 2 b  ab2  b3  b2 c  bc 2 
c 3
 c 2 a  ca 2 t
AM GM
3 3 a3 ˜ a 2 b ˜ ab2  3 3 b3 ˜ b2 c ˜ bc 2  3 3 c3 ˜ c 2 a ˜ ca 2
3 a 2 b  b 2 c  c 2 a , de
unde a2  b2  c2 t a 2 b  b2 c  c 2 a. Acum, revenind la enunț,
a
2 2
2
b c 2 2
ab  bc  ca a  b  c2 ab  bc  ca
2 2

 n˜ 2 t  n˜ 2
a bb cc a
2 2 2
a b c 2 2
a b c
2 2 2
a  b2  c 2
ab  bc  ca ab  bc  ca
a 2  b2  c 2  n ˜ 2 , deci este suficient să arătăm că a 2  b2  c 2  n ˜ 2 t n3.
a b c2 2
a  b2  c 2
9t ab  bc  ca
Notăm, t a 2  b 2  c 2 Ÿ ab  bc  ca . Astfel, a 2  b2  c 2  n ˜ 2 2 2 t n  3
2 a b c
9t
œ t  n˜ t 3  n œ 2t 2  9n  nt t 6t  2nt œ 2t 2  3t n  2  9n t 0 œ t  3 2t  3n t 0.
2t
Ultima inegalitate este adevărată deoarece 9 a  b  c d 3 a 2  b2  c 2 Ÿ a 2  b2  c 2 t 3, iar
2

2t t 6 t 3n œ 0 d n d 2 .

b n 1  c n 1 c n 1  a n 1 a n 1  b n 1
L:573. Dacă a, b, c > 1 şi n N * , să se arate că log a  log b  log c t 3n  3.
b2 c 2 c2 a2 a 2 b 2
Gheorghe Ghiţă, Buzău
Rezolvare: Vom stabili mai întâi următoarea inegalitate:
n 1
a n 1  b n 1 § a 2  b 2 ·
Dacă a, b, c > 0, şi n N , atunci: t ¨¨ *
¸ .
¸
a 2 b 2 © a b ¹
Demonstraţie. Pentru n = 1, inegalitatea este evidentă. Fie nN , n t 2.
In inegalitatea lui Hölder, pentru două perechi de numere reale pozitive,
n 1 n 1
a2 b2 n ,y n ,p
n
x1 , x2 ; y1 a b n, q , avem:
n 1 n 1 2
n 1
a n b n
1 n 1
ª§ 2 n
· § 2 ·
n
º n ª n 1 n n
º n
» «§¨ n ·¸ n 1 §¨ n ·¸ n 1 »
n 1 n 1 n 1
«¨ a ¸ ¨ b ¸ a2 b2
¸  ¨ n 1  b t ˜a  ˜b œ
n n
«¨ n 1 ¸ » «¨ a ¸ ¨ ¸ » n 1 n 1
«¨© a n ¸ ¨ n
¹ ©b
¸
¹ » « © ¹ © ¹ » a n b n
¬ ¼ ¬ ¼
n 1 n 1
a n 1  b n 1 § a 2  b 2 ·
a œ
1
n 1 n
 b n1 n (a  b) n t a 2  b 2 Ÿ a n1  b n1 (a  b) n1 t a 2  b 2
t ¨¨ ¸¸ .
a 2 b2 © a b ¹
Scriem relaţiile analoage, aplicăm logaritmul în bazele supraunitare c, b, a, şi prin adunarea
n 1

¦ a n 1  b n 1
¦ ¦ ¦log
§ a 2 b2 · a 2 b2
lor, avem: logc logc ¨¨ ¸¸
t (n 1) logc t (n 1) ab
© a b ¹ a b c
a 2 b2
n 1
2 ¦
logc ab
n 1
2 ¦
logc a  logc b t n21˜6 3n 3.
În ultimele trei inegalităţi s-au aplicat inegalităţile:
x2  y 2 x y
t xy ; x, y ! 0,  t 2; x, y ! 0.
x y y x
1 1 1 1
L:574. Arătaţi că în orice triunghi nedreptunghic ABC avem:   ! .
2  tg A 2  tg B 2  tg C 2
2 2 2

Neculai Stanciu, Buzău și Titu Zvonaru, Comănești


- PROBLEME PROPUSE -

Rezolvare: Notând x tgA, y tgB, z tgC , inegalitatea dată devine:


1 1 1 1
  t .
2 x 2
2 y 2
2 z 2
2
Eliminând numitorii și ţinând cont de faptul că x  y  z xyz , inegalitatea este ehivalentă succesiv cu
16  4 x 2  4 y 2  4 z 2 t x 2 y 2 z 2 œ 16  4( x 2  y 2  z 2 ) t ( x  y  z ) 2
œ 16  3( x 2  y 2  z 2 )  2( xy  yz  zx) t 0 œ 16  x 2  y 2  z 2  ( x  y) 2  ( y  z) 2  ( z  x) 2 t 0 ,
adevărată.
Inegalitatea este strictă. Egalitatea are loc dacă triunghiul ABC este degenerat, adică două dintre
unghiuri sale sunt drepte și unul este nul.

L:575. Să se rezolve în ecuația 8sin x ˜ cos2 x 5 și să se arate că ecuația are o infinitate de


soluții x n , cu n  .
0

Ionel Tudor, Călugăreni, Giurgiu


Rezolvare:
Evident sin x² 0 Ÿ x  2kS ;(2k  1)S , k  . Ecuația dată este echivalentă cu 4sin 2 x ˜ cos x 5.
§S ·
Dacă x  ¨  2kS ; S  2kS ¸ Ÿ cos x¢0 și sin 2 x 2sin x cos x¢0 , deci sin 2 x ˜ cos x²0 . Domeniul de
©2 ¹

existență al soluțiilor reale ale ecuației este D 2kS ; (2k  1)S \ §¨ ­®¯S2  22kkS ½¾¿ ·¸ 
k © kk ¹
5 5
Dacă ecuația inițială o scriem ca 2sin 2 x cos x Ÿ sin 3x  sin x . Sau mai departe putem
2 2
scrie 8sin 3 x  8sin x  5 0 . Notând y 2sin x  0; 2 și ecuația devine y3  4 y  5 0 Ÿ

5  4( y 
3
y3  5) 0 Ÿ ( y  5) y 2  y 5  1 0 Ÿ y 2  y 5  1 0 cu soluțiile:

5 r1 ­
° 5 1 ½
°
y1,2  0; 2 . Cum y 2sin x Ÿ S1 ®(1) arcsin
k
 kS k  ¾,
2 °
¯ 4 °
¿
­
° 5 1 ½
°
S2 ®(1) arcsin
k
 kS k  ¾ . Mulțimea soluțiilor reale ale ecuației este S S1 ‰ S2 .
°
¯ 4 °
¿
Pentru a demonstra existența infinității soluțiilor ecuației de tipul x n0 , cu n  , vom considera cazurile
pentru k par și impar după ce scriem S1 ^(1) k
˜180  1800 ˜ k k  ` și
S2 ^(1) k
˜ 540  1800 ˜ k k  `.
S 5 1
S-a folosit faptul că cos cos 360 Ÿ
5 4
62 5 5 1 5 1
cos 720 2cos 2 360  1 2 ˜ 1 sin180 Ÿ arcsin 180 , iar
16 4 4
5 1 S 5 1
arcsin  arccos 900  360 540.
4 2 4
Pentru k par: k 2m  Ÿ x1 1 180  1800 ˜ 2m (20m  1) ˜180 , și
x2 540  1800 ˜ 2m (20m  3) ˜180 , cu m natural;
Pentru k impar: k 2m  1 Ÿ x1' 180  1800  1800 ˜ 2m (20m  9) ˜180 , iar
x2' 540  1800  1800 ˜ 2m (20m  7) ˜180 , cu m natural;
-PROBLEME PROPUSE -

L:576. a) Să se determine a0 , a1 ,......, an  pentru care


(1  x  nx)(1  x) n 1
a0  a1 x  a2 x  .......  an x , n  și pentru orice x 
2 n
\ ^1` .

b) Calculați suma: S 1  2 ˜ 2C 1n  3 ˜ 22 Cn2  4 ˜ 23 Cn3  ........  (1)n (n  1)2n Cnn .


Ionel Tudor, Călugăreni, Giurgiu
Rezolvare:
a) Fie f ( x) (1  x  nx)(1  x)n1 (1  x)n  nx(1  x)n1 . Folosind binomul lui Newton, membrul
n n n
stâng se scrie restrâns astfel: ¦C
k 0
k
n x k  ¦ kCnk x k
k 0
¦ (k  1)C
k 0
k
n x k și identificând cu membrul drept se

găsesc coeficienții ak (k  1)C  , k k


n 0, n.
0
b) Din a) rezultă 1  ˜2C x  3C x  ..........  (n  1)Cnn x n
1
n
2 2
n (1  x  nx)(1  x) n1, n  , x  \ ^1`.
Pentru x= - 2 se obține S (1)n (2n  1), n  .

ƒ Clasa a XI-a
3S 3 21S 7
L:577. a) Dovediți inegalitățile: cos ² ; cos ¢ ;
16 4 100 8
S S n 2
b) Rezolvați în * ecuația sin  cos . Ionel Tudor, Călugăreni, Giurgiu
2n 2n 4
Rezolvare:
3S 3S S ª Sº 3S S 5 1 3
a) Cum 0¢ ¢ ¢ și funcția cosinus e descrescătoare pe «¬0; 2 »¼ Ÿ cos 16 ² cos 5 ² .
16 15 2 4 4
§ S· §S S·
b) Cu formula sin a  cos a 2 ˜ cos ¨ a  ¸ , ecuația devine: 4 2 cos ¨  ¸ n 2˜ 2Ÿ
© 4¹ © 2n 4 ¹
§S S ·
8cos ¨  ¸ x  2, (*) , unde s-a notat 2n cu x; Evident, x trebuie să un număr par din mulțimea
© x 4¹
§S S ·
> 4;36@ ; Fie f : > 4;36@ o, f ( x) 8cos ¨  ¸  x  2 derivabilă cu derivata
© x 4¹
8S §S S · 1
f I ( x) 2
sin ¨  ¸  ¢0 Ÿ f este strict descrescătoare pe > 4;36@ . Cum
x © x 4¹ 2 x
f (4) ˜ f (36)¢0 Ÿ x0  > 4;36@ unic , astfel încât f ( x0 ) 0 (conform Proprietății lui Darboux ). Se găsește
x = 20 adică n= 10.

3(1 x )
L:578. Rezolvați ecuația 3 2
˜ x 1. Marin Chirciu, Pitești
3( x 1)
2
3
Rezolvare: Evident x trebuie să fie pozitiv. Ecuația dată este echivalentă cu 1. Fie funcția
x
3( x 1)
3( x 1 3x
2
) 3 2
( ln 3  1)
3 2
f : (0; f) o , f ( x) având derivata f I ( x) . Cum
x x2
lim f ( x) f, lim f ( x) f, din tabelul de
x of x o0
x² 0
1 § 2 ·
variație al funcției se observă că ecuația f(x)=1 are cel mult două soluții. Acestea sunt x1  ¨ 0; ¸ și
3 © 3ln 3 ¹
§ 2 ·
x2 1 ¨ ,f¸.
© 3ln 3 ¹
L:579. Determinaţi funcţiile continue f : R o R pentru care
f ( x  y) f ( x)  f ( y)  xy ( x  y)( x 2  xy  y 2 ), x, y  R.
Gheorghe Ghiţă, Buzău
Rezolvare. Înmulţim relaţia dată cu 5 şi aceasta devine:
5 f ( x  y) 5 f ( x)  5 f ( y)  5xy ( x 3  2 x 2 y  2 xy 2  y 3 ) œ
5 f ( x  y) 5 f ( x)  5 f ( y)  5x 4 y  10 x 3 y 2  10 x 2 y 3  5xy 4 œ
5 f ( x  y ) 5 f ( x )  5 f ( y )  ( x  y ) 5  x 5  y 5 œ 5 f ( x  y )  ( x  y ) 5  5 f ( x)  x 5  5 f ( y )  y 5 .
Notăm g ( x) 5 f ( x)  x 5 , g : R o R şi atunci ultima relaţie devine:
g ( x  y) g ( x)  g ( y), x, y  R (ecuaţia funcţională Cauchy)
Cum funcţia f este continuă rezultă că şi funcţia g continuă şi cum g verifică ecuaţia de mai sus,
ax  x 5
atunci g ( x ) ax, aR * . Funcţia dată este f ( x) , a  R*.
5
1 n ai2 e ai
L:580. Dacă ai  , i  ^1,2,..., n`, atunci să se demonstreze că şirul x n ¦ este mărginit.
n i 1 ai2  e ai
Constantin Rusu, Râmnicu Sărat
2 ai
2a e
Rezolvare: Cu inegalitatea MH-MG avem i
d ai2 e ai , i 1, n , (1).
a e 2
i
ai

Considerăm funcţia f : R o R, f ( x) x 2 e x . Deoarece,


x2 L 'H 2x L 'H 2
lim f ( x) lim lim lim 0 , rezultă că y 0 este asimptotă orizontală
x of x of e  x x of  e  x x of e  x

la  f .
Funcţia f este continuă şi derivabilă deoarece este produs de funcţii elementare continue şi derivabile. Avem
f c( x) e x (2 x  x 2 ) şi f c( x) 0 œ x 0 sau x 2 . Rezultă următorul tabel de variaţie
x f 2 0 f
f c(x) +++++++++++++++++0-----------------------0+++++++++++++++++++++++++++
f (x) 4
0 0 f
e2
4 4 2
Deducem că f are un maxim absolut egal cu 2
. Deci, x 2 e x d 2 œ x 2 e x d , x  R , (2).
e e e
2ai2 e ai 2
Din (1) şi (2) obţinem d , care sumate pentru i 1, n conduc la
ai  e
2 ai
e
n
ai2 e ai n § 1·
¦
i 1 ai  e
2 ai
d œ xn  ¨ 0, ¸ , i.e. concluzia dorită.
e © e¹


n n
3x  x 2  2  3x  x 2  2
L:581. Pentru orice n  , n t 2 , să se calculeze lim n
;
x of x
Adrian Stan, Buzău
Rezolvare:
- PROBLEME PROPUSE -

2 n 2 ª 2 2 º
(3x  x 1  x n ˜ «(3  1  2 ) n  (3  1  2 ) n »
)  (3x  x 1  2 ) n
x 2
x ¬ x x ¼
L lim n
lim 4n  2n .
x of x x of n
x
L:582. Fie A o matrice pătratică de ordinul 2 cu proprietățile det( A  3I 2 ) 4 și det( A  2 I 2 ) 9 . Să
se calculeze ( A  I 2 )2018 . Adrian Stan, Buzău
Rezolvare:
§a b· §a 3 b ·
Fie A ¨ ¸  M 2 Ÿ A  3I 2 ¨ ¸ Ÿ det( A  3I 2 ) 4 œ (a  3)d  3)  bc 4 .
©c d¹ © c d  3¹
§a 2 b ·
A  2I 2 ¨ ¸ Ÿ det( A  2 I 2 ) 9 œ (a  2)(d  2)  bc 9 . Scăzând ultimele două relații se
© c d  2¹
obține că a  d 2 și ad  bc 1. Din relația A2  (a  d ) A  (ad  bc) I 2 O2 se obține prin înlocuire
O2 Ÿ A  I 2
2018
că A  2 A  I 2 O2 œ ( A  I 2 )2
2
O2 .

ƒ Clasa a XII-a
S tgx
L:583. Să se determine primitivele funcției: f : (0; ) o , f ( x) .
2 1  cos3 x
Marin Chirciu, Pitești
sin x 1
Rezolvare : ³ f ( x)dx ³ cos x(1  cos 3
x)
dx ³ cos x(1  cos 3
x)
˜ ( cos x) I dx

1 1
³ cos x˜(  ) ˜ (cosx) I dx
2

cos x 1  cos3 x
3

(cos x) I cos 2 x 1
³ dx  ³ (cos x) I dx  ln(cos x)  ln(1  cos3 x)  C .
cos x 1  cos x
3
3
1
2
4 ˜ (2 x  1) 2016
L:584. Să se calculeze integrala I ³0 (2 x  1)2018 dx . Ionel Tudor, Călugăreni, Giurgiu

( dată la Concursul interjudețean “Laurențiu Panaitopol”- Giurgiu, 2017)


Rezolvare:
1

4 ˜ (2 x  1) 2016 2x 1
2 0
1 4dx
³0 (2 x  1)2018 dx ³t Ÿ dt
2016
I dt . S-a folosit substituția t .
1
2017 2x 1 (2 x  1) 2

L:585. Să se rezolve în ecuațiile: a) 64 x4  64 x3  8x  1 0;


b) x4  8x3  24 x2  32 x  65092608 0. Ionel Tudor, Călugăreni, Giurgiu

Rezolvare:
a) 64 x4  64 x3  8x  1 0 œ 64 x4  16 x 2  1  64 x3  16 x 2  8x 0 œ (8x 2  4 x  1)2 0.
­
°1 r 3 ½
°
Din 8 x 2  4 x  1 0 Ÿ x  ® ¾.
° 4 ¿
¯ °
b) Se face substituția x 2 y rezultând 16 y 4  64 y3  96 y 2  64 y  65092608 0 œ
( y  1)4 4068289 20172 œ ¬ª( y  1)2  2017 ¼º ˜ ¬ª( y  1)2  2017 ¼º 0 cu soluțiile
- PROBLEME PROPUSE -

^ ` ^
y  1 r 2017, 1 r i 2017 , de unde, x  2 r 2 2017, 2 r i 2017 . `
L:586. Dacă x1 , x2 , x3 sunt rădăcinile ecuației x3  x  a 0, a  * , calculați suma

1  x1 1  x2 1  x3
S   . Marin Chirciu, Pitești
1  x1 1  x2 1  x3
1 x
Rezolvare: Cu substituția t suma căutată este S t1  t2  t3 , unde t1 , t2 , t3 sunt rădăcinile ecuației
1 x
4  3a
at 3  (3a  4)t 2  (3a  4)t  a 0; Rezultă S .
a
L:587. Să se arate că ecuația x2 n1  Cn1 x2 n1  Cn2 x2 n3  .....  Cnn1 x3  2Cn n x  m 0 are cel puțin o
rădăcină nulă, m fiind un parametru real.
Petre Rău, Galați
Rezolvare: Știm că x1  x2  x3  ....  x2 n1 0 și x1 ˜ x2 ˜ x3 ˜ .... ˜ x2 n1 m . Pentru m z 0 Ÿ ecuația nu are

«¬
rădăcini nule și poate fi scrisă de forma x ª x 2  1  1º
n

»¼ m. Atunci, cel puțin o rădăcină a ecuației date


este pozitivă și cel puțin o rădăcină este negativă. Fie, de exemplu, x1 rădăcina pozitivă iar x2 rădăcina
negativă. În acest caz din ecuația restrânsă, pentru x1 pozitiv rezultă că m este negativ, iar din x2 negativ
rezultă că m este pozitiv. În concluzie rezultă că m=0, ceea ce înseamnă că ecuația dată are cel puțin o
rădăcină nulă.

L:588. Dacă ecuaţia (1): ax 4  bx 3  cx 2  dx  e 0 (unde a, b, c, d , e  R * ) are rădacini reale


distincte în progresie aritmetică, atunci, determinaţi rădăcinile ecuaţiei
(2): 4ax 3  3bx 2  2cx  d 0 , în funcţie de rădăcinile ecuaţiei (1) şi arătaţi că sunt tot în progresie
aritmetică.
D.M. Bătineţu-Giurgiu, București și Neculai Stanciu, Buzău
Rezolvare: Fie x1 , x2 , x3 , x4 soluţiile ecuaţiei f ( x) ax 4  bx 3  cx 2  dx  e 0 .
x1  x 4 x 2  x3 (1)
Din ipoteză rezultă k.
2 2
4
f ( x) a– ( x  x k ) a( x 2  2kx  m)( x 2  2kx  n) , unde am notat m x1 x4 şi n x2 x3 . Deoarece
k 1

mn
4ax 3  3bx 2  2cx  d f c( x) avem că ecuaţia (2) f c( x) 4a( x  k )( x 2  2kx  ) 0 are
2
x1  x4 x1  x4 x1  x4
soluţiile: y1  x12  x42  2 x2 x4 ; y 2 ; y3  x12  x42  2 x2 x4 .
2 2 2
y  y3
Deoarece 1 y 2 rezultă că şi rădăcinile ecuaţiei (2) sunt în progresie aritmetică.
2
ª Sº
L:589. Fie f , g , h : «0; » o , f ( x) cos 4 x, g ( x) sin 4 x, h( x) 2sin 2 x ˜ cos 2 x. Să se calculeze
¬ 2¼
³ > f ( x)  g ( x)  3h( x)@dx . Adrian Stan, Buzău
Rezolvare: f ( x)  g ( x)  3h( x) cos4 x  sin 4 x  6sin 2 x cos2 x (cos2 x  sin 2 x)2  8sin 2 x cos2 x
sin 4 x
1  2sin 2 2 x cos 4 x . Atunci, ³ > f ( x)  g ( x)  3h( x)@dx ³ cos 4 xdx  C.
4
L:590. Să se determine funcţia derivabilă f : R o R cu proprietatea că :
- PROBLEME REZOLVATE -

³
f ( x ) sin x  e t f ( x  t )dt,x R şi f (0) 0.
0
Gheorghe Ghiţă, Buzău

Rezolvare : Cu schimbarea de variabilă x t u, avem:

0 x x

f ( x ) sin x  e
³
x
u x
f (u )du,xR œ f ( x ) sin x  e x
³ e f (u)du,xR œ e f ( x)
0
u x x
³
e sin x  e u f (u )du,x  R.
0

Prin derivare, obţinem: e f ( x)  e f ( x) e sin x  e cosx  e f ( x),xR œ f ( x ) sin x  cosx,xR.


x x ' x x x ,

Prin integrare, rezultă că f ( x) sin x cosx C.


Din condiţia f (0) 0, rezultă C 1, şi atunci funcţia cerută este f ( x) sin x cosx 1.

e x cos x  sin 2 x
³e ˜ dx , x  R .
x
L:591. Să se calculeze Constantin Rusu, Râmnicu Sărat
(e x  sin x) 2
e x cos x  sin 2 x
Rezolvare: ³e ˜
x
dx
(e x  sin x) 2
e x cos x  e x sin x  sin 2 x  sin x cos x  e x sin x  sin x cos x
³e ˜
x
dx
(e x  sin x) 2
e x (sin x  cos x)(e x  sin x)  e x sin x(e x  cos x)
³ (e x  sin x) 2
dx

c
(e x sin x)c(e x  sin x)  e x sin x(e x  sin x)c § e x sin x · e x sin x
³ (e x  sin x) 2
dx ³ ¨¨© e x  sin x ¸¸¹ dx e x  sin x
C .

L:592. Dacă f : >0,1@ o este o funcție integrabilă, aratăți că are loc inegalitatea:
4
1
§1 · 1

³ f x  ¨ ³ f t dt ¸ d 2 ˜ ³ f 2 x dx.
4
Florin Stănescu, Găești, Dâmbovița
0 ©0 ¹ 0

Rezolvare: Dacă x t 0, cu ajutorul inegalitatii C-B-S, putem scrie:

1˜ d 1  1 ª« 2 x  1  x 2 º»
2 2
2 1  x , de unde ,
2 2
2 x  1 ˜ 1  x2 2 1  2 x  x2
¬ ¼
x2  1 x2  1
2 x  1 ˜ 1  x 2 d 2 1  x 2 1  x Ÿ x 
2
d x 1 Ÿ d 1 x  x .
2 2
1  x4
2
Acum, înlocuind pe x cu x , rezultă d 1  x  x2 .
2
1
Dacă ³ f x dx
0
0, atunci inegalitatea din enunț este evidentă.

1
f x
Presupunem că ³ f x dx z 0. În ultima inegalitate, punând 1
, în loc de x, și apoi integrând

³ f t dt
0

0
inegalitatea rezultată, se obține cerința problemei.
- PROBLEME PROPUSE -

„Oamenii pot fi făcuți să urmeze o cale de acțiune,


dar se poate să nu fie făcuți și pentru a o înțelege.”
Confucius
(551- 479)

4. Probleme propuse

ƒ Clasa a V-a
G:772. Punctul M aparţine segmentului > AB @ şi > MB @ este de 23 ori jumătatea treimii sfertului
segmentului > AB @ . A câta parte din > AB @ este > AM @ ? Ion Stănescu, Smeeni, Buzău

G:773. Precupeața Ioana vinde în piață ouă. Ea reușește să vândă întreaga cantitate în 4 zile după cum
urmează:
În prima zi vinde din cantitatea totală de ouă și dintr-un ou,
În a doua zi vinde din cantitatea rămasă după prima zi și dintr-un ou,
În a treia zi vinde din cantitatea rămasă după a doua zi și dintr-un ou,
În cea de-a patra zi vinde din cantitate după a treia zi și dintr-un ou ramânând astfel fără niciun ou după
cele patru zile. Câte ouă a avut precupeața ințial?
Ramona- Carmen Grigore, Maria Boborel , Craiova

2
G:774. Determinați numărul natural ab știind că aa  aa u ab  b  b 2018 .
Nicolae Ivășchescu , Canada
2018 n  2 2018 n  6
G:775. Fie numărul x 2 ˜5  7, n  *
. Să se afle n știind că: suma cifrelor numărului x este
36348.
IulianaTrașcă, Olt
G:776. Să se scrie numerele 38, 38 , 38 , n  ca sumă de trei pătrate perfecte nenule și distincte.
2 n

Marin Chirciu, Pitești


G:777. Scrieți numărul 1035 ca diferență de două pătrate perfecte.
Diaconu Radu, Sibiu, Sclipirea Minții, 21/2018
G:778. Determinați valorile naturale ale lui n pentru care numărul a 0,7  1,7  2,7  ....  n,7 este, de
asemenea natural.
Marian Ciuperceanu , Craiova
G:779. Să se determine cifrele a şi b ştiind că în sistemul zecimal restul împărţirii numărului aaaa la ba
este ab .
Petre Rău, Galați
G:780. Determinați numerele prime și astfel încât p  q și p  q să fie de asemenea numere prime.
4 4

Lucian Tuțescu , Grigorie Dan – Lucian, Craiova


G:781. Arătaţi că numărul N 111 ...

1 este compus.
2010 de1
D.M. Bătineţu-Giurgiu, Bucureşti şi Neculai Stanciu, Buzău
G:782. Să se arate că numărul 23n2 ˜ 32n1  7n este divizibil cu 13 pentru orice număr natural n.

Gheorghe Ghiţă, Buzău


- PROBLEME PROPUSE -

G:783. Determinați câtul și restul împărțirii numărului 72018 la 8 ˜ 72015 IulianaTrașcă, Olt
G:784. Notăm cu A mulţimea numerelor de cinci cifre distincte formate cu elementele mulţimii ^1,2,3,7,8`.
Determinaţi m  A astfel încât 4m  A .(În legătură cu problema E:14596)
Titu Zvonaru, Comăneşti şi Neculai Stanciu, Buzău
G:785. a)Determinaţi cel mai mic număr natural de forma a 2  b 2 care are 25 de divizori cu proprietatea că
a şi b sunt numere naturale, prime între ele.;
b) Determinaţi toate perechile (a, b) de numere naturale, prime între ele, astfel încât a 2  b 2 este cel mai mic
număr care are 25 de divizori.
Marius Drăgan, Bucureşti şi Neculai Stanciu, Buzău
n k
G:786. Să se demonstreze că 10  10 se poate scrie ca sumă a 18k pătrate perfecte nenule oricare ar fi
n

n şi k numere naturale nenule.


Marius Drăgan, Bucureşti şi Neculai Stanciu, Buzău
46391
G:787. Să se arate că fracția F este reductibilă.
34289
Ionel Tudor , Călugăreni , Giurgiu

ƒ Clasa a VI-a
x 1 3 z
G:788. Aflaţi x, y, z, întregi, dacă . Ion Stănescu, Smeeni, Buzău
2 y x 1
G:789. Găsiți cel mai mare și cel mai mic număr natural cu cifre diferite, astfel ȋncȃt cifra miilor să fie7, iar
produsul cifrelor din stȃnga lui 7 să fie maxim și egal cu produsul cifrelor din dreapta lui 7.
Petre Păunescu, Roșiorii de Vede, Teleorman
G:790. Să se arate că numărul m=4072324 este pătrat perfect iar numărul n=1357441 este compus.
Ionel Tudor , Călugăreni , Giurgiu
G:791. Determinați câte regiuni determină 2019 dreptre coplanare, oricare două neparalele și oricare trei
neconcurente.
Nicolae Ivășchescu, Canada
G:792. Rezolvați în numere întregi ecuația: x  7 xy 11.
Radu Diaconu, Sibiu
G:793. Determinați numerele naturale x, y și z știind că satisfac simultan condițiile:
x y z
1) ; 2) xyz 2 xy  xz  5 yz ; Mircea Mario Stoica, Arad
5 2 4

G:794. Fie a, b, c  , pentru care există numerele prime între ele x și y cu x  y n , unde n  astfel
(2a  b)(c  2a)
încât 2na xb  yc . Arătați c㠏 . Marin Chirciu, Pitești
xy
G:795. Să se arate că pentru orice k natural, k≥1, numărul A = 9k+19k +29k +…+99k se divide cu 540 atunci
când k este impar și cu 30 atunci când k este par.
Petre Rău, Galați
G:796. Să se arate că oricare ar fi cifra nenulă x, numărul A 20812018 x  8102 x 2081  2083 x 2091 este
divizibil cu zece.
IulianaTrașcă, Olt
G:797. Demonstraţi că numărul 999
9  199
 ...  ...

9 00
 ...

0 este compus.
2011de 9 1005de 9,1005de 0
D.M. Bătineţu-Giurgiu, Bucureşti şi Neculai Stanciu, Buzău
______ ab  ac  b ac  ad  2c ab  ad  b  2d
G:798. Să se determine numerele abcd , ştiind că: .
2a  3 3a  3 4a 2  5a
- PROBLEME PROPUSE-

Gheorghe Ghiţă, Buzău

G:799. Trei copii au împreună 410 lei. După ce fiecare cheltuie din banii proprii 75% , 80% respectiv
85%, atunci rămân cu sume egale de bani. Câți bani a avut fiecare la început?
Marian Ciuperceanu , Craiova
G:800. Stabiliţi paritatea numărului A n  2011 3n  5 n  2018  7n , unde n .
IulianaTrașcă, Olt
G:801. a) Fie k şi m numere naturale nenule. Arătaţi că dacă A (n1  1) ˜ (n2  1) ˜ ... ˜ (nk  1)  1 şi
B (nk 1  1) ˜ (nk 2  1) ˜ ... ˜ (nk m  1)  1, atunci AB  A  B (n1  1) ˜ (n2  1) ˜ ... ˜ (nk m  1)  1 .
b) Pe o tablă sunt scrise inversele numerelor 1, 2, 3,…, 2017. Alegem două dintre numerele de pe tablă, le
ştergem şi în locul lor scriem numărul obţinut prin adunarea sumei lor cu produsul lor. Continuăm procedeul
până când pe tablă rămâne un singur număr. Care este ultimul număr rămas pe tablă?
Marius Drăgan, Bucureşti şi Neculai Stanciu, Buzău
G:802. Dacă două şiruri a1 , a2 ,..., a2013 şi b1 , b2 ,..., b2013 sunt construite după regulile :
a n 1  a n  2
an a
(i) a1 1999 şi a2 2003 ;(ii) b1 2011 şi b2 2017 ;(iii) n ! 2 şi fiecare fracţie n se
bn 1  bn  2
bn bn
a
consideră ireductibilă, atunci determinaţi cea mai mare şi cea mai mică fracţie de forma n .
bn
Marius Drăgan, Bucureşti şi Neculai Stanciu, Buzău
x z x z x z
G:803. Să se arate că există o infinitate de fracții și cu x, y, z, t  * astfel încât  ˜
y t y t y t
Viorica Dogaru , Giurgiu

ƒ Clasa a VI-a
G:804. Rezolvaţi, în R, ecuaţia . Ion Stănescu, Smeeni, Buzău

G:805. Fie a, b ! 0 astfel încât a 3  b.3 2018ab. Arătați că: a  b d 2018.


Diaconu Radu, Sibiu,
1
G:806. Numerele reale strict pozitive a0 , a1 , a2 ,... verifică relaţia a n 1 a n  , oricare ar fi numărul
an
natural n . Să se demonstreze că a1985 , a1986 ,…, a 2047 sunt mai mari decât 63.
Marius Drăgan, Bucureşti şi Neculai Stanciu, Buzău

G:807. Să se arate că numărul 284 n se scrie ca o sumă de 8 pătrate perfecte pentru n număr impar şi ca o
sumă de 64 de pătrate perfecte pentru n număr par.
Constantin Rusu, Râmnicu Sărat
81n5 9n3 n
G:808.Să se arate că pentru orice număr natulal n, numărul   este natural.
40 8 10
Petre Rău, Galați
G:809.Rezolvați în mulțimea numerelor întregi ecuația x  y  65x  21y  1166 0 .
2 2

Mircea Mario Stoica, Arad


G:810.Determinați câte numere naturale mai mici decât 999999 , se pot scrie în forma n4  4n3  6n2  4n ,
unde Care este cel mai mare dintre ele ? Ionel Tudor , Călugăreni , Giurgiu

G:811.Rezolvați ecuația 6  x 5  7  2 10 2 1. Nicolae Ivășchescu, Canada


- PROBLEME PROPUSE -

2017 ˜ 2018  2017 ˜ 2018  2017 ˜ 2018


G:812. Arătați că ²1 .
2016 ˜ 2017  2016 ˜ 2017  2016 ˜ 2017
Marian Ciuperceanu , Craiova
7 12 n 3
2
G:813. Să se calculeze partea întreagă a numărului:  ... , nN , n t 2.
1˜ 2˜3 2˜3˜ 4 (n 1)n(n 1)
Gheorghe Ghiţă, Buzău
G:814. Aflaţi perimetrul triunghiului ∆ABC , ştiind că m( A) 1050 , m( B) 150 , AC 2 3 cm.
Badea Daniela, Ploiești, Prahova
G:815. Determinați aria trapezului ABCD, cu AB CD , AB 64 cm, BC 40 cm, CD 14 cm și
DA 30 cm .
Marian Ciuperceanu , Craiova
G:816. Dacă un triunghi oarecare este asemenea cu triunghiul format de medianele sale atunci determinaţi
raportul de asemănare.
Nela Ciceu, Roşiori, Bacău şi Roxana Mihaela Stanciu, Buzău

ƒ Clasa a VIII-a

G:817. Rezolvați ecuația 5x2  10 y 2  10  14 xy  10 x  12 y 0. Nicolae Ivășchescu, Canada

G:818. Dacă x rezolvaţi ecuaţia 3 + [2x+3] = 13, unde x , respectiv, > x @ reprezintă modul
din x respectiv partea întreagă a lui x. Ion Stănescu, Smeeni, Buzău

G:819. Știind că inegalitatea 1895x( x 2  4) 1004( x  2) t 0, este adevărată, atunci să se arate că dacă a, b ! 0,
§ a3 b3 · §a b·
atunci are loc inegalitatea 1895¨¨ 3  3 ¸¸ t891¨  ¸  2008. Gheorghe Ghiţă, Buzău
©b a ¹ ©b a¹
G:820. Determinaţi toate tripletele de numere reale nenule (a, b, c) , care verifică relaţiile:
2 2 2
§b· §c· §a·
a ¨ ¸ b¨ ¸ c¨ ¸ 1. Marius Drăgan, Bucureşti şi Neculai Stanciu, Buzău
©c¹ ©a¹ ©b¹
G:821. Pentru ce valori naturale ale lui a expresia E a1010  a1009  a 2  a se divide cu 2018 ?

Petre Rău, Galați


G:822. Cu formule de calcul prescurtat, descompuneți ȋn factori:
2ab  3cd 2abcd
A   1, a, b, c, d ² 0, reale. Petre Păunescu, Roșiorii de Vede, Teleorman
6 3
G:823. a) Să se arate că există numerele naturale nenule x și y , cu x>y astfel încât
n(n  1)(n  2)
x2  y 2 (k  1)3  k 3 , k  * și deduceți suma 12  22  32  ...  n2 , n  *.
6
12  22  32  ...  n 2
b) Determinați n  * , pentru care , este număr natural.
1  2  3  ...  n
Ionel Tudor , Călugăreni , Giurgiu
- PROBLEME PROPUSE-

G:824. Fie x, y² 0 și x3  y 3 x  y . Să se arate că: x ˜ y¢1 și x 2  y 2 ¢1 .


Alina Tigae, Simona Miu , Craiova
G:825. Să se arate că:
a  1 a(a 2  1) a  1 a3  1 a3  1 a4 1
i) t 3 , a ! 0 ; ii) d 2 , a ! 0 ; iii) 2 t a2  a 1 t 4 , a ! 0 .
2 a 1 2 a 1 a 1 2
D.M. Bătineţu-Giurgiu, Bucureşti şi Neculai Stanciu, Buzău
G:826. Dacă x, y t 0 și x3  y3  ( x  y)3  18( x  y)  27 xy 1620, calculați x  y.
Marin Chirciu, Pitești
G:827. Rezolvați, în numere reale, inecuația: x  2 x  x  2 ¢ 0 .
4 3

Marian Ciuperceanu , Craiova


a  1 a3  1
G:828. Să se demonstreze că d 2 , a ! 0 .
2 a 1
Nela Ciceu, Roşiori, Bacău şi Roxana Mihaela Stanciu, Buzău
G:829. În paralelipipedul dreptunghic ABCDA' B' C ' D' , notăm AB a, BC b, AA' c. Știind că latura
S[ BDD' ] 1  17
c verifică ecuația 4 x 3  5x  2 0 , arătați că: t , unde S[BDD' ] este aria
S[ ABCD ] 4 2
triunghiului BDD' , iar S[ABCD ] este aria dreptunghiului ABCD. Radu Diaconu, Sibiu

ƒ Clasa a IX-a
L:593. Fie m un parametru întreg astfel încât ecuaţia x 2  mx  m  2010 0 are o rădăcină număr întreg.
Rezolvaţi ecuaţia şi determinaţi parametrul m .
Nela Ciceu, Roşiori, Bacău şi Roxana Mihaela Stanciu, Buzău
L:594. Să se demonstreze că ( pq  1) 2 t 16( p  1)(q  1) , oricare ar fi p, q t 3 .
Neculai Stanciu, Buzău, şi Titu Zvonaru, Comăneşti.
L:595. Să se determine cel mai mare număr întreg a, așa încât ecuația x3  4 x 2  (4  a) x  2a 0 să aibă
o rădăcină întreagă și celelalte nereale. Petre Rău, Galați
4
14  24  34  ....  n4 § n  1 ·
L:596. Să se arate că: t¨ ¸ , n t 1. Ionel Tudor , Călugăreni , Giurgiu
n © 2 ¹
L:597. Fie (bn )nt1 o progresie geometrică crescătoare cu termenii pozitivi. Demonstrați că
b2n  b1 t (2n  1)(bn1  bn ), n  *
. Marin Chirciu,Pitești
5 5
R R care verifică relația f(x)+ f(2018-x)=
L:598. Fie funcția f:R x  , oricare ar fi x  .
2018 2
Arătați că f(x)+f(2018-x)=12 oricare ar fi x R . Determinați funcția f.
Constantin Dinu, Buzău
š 24 2
L:599. În triunghiul ABC , se dau a  b 2 2 , b t c și m( A) 162 D . Să se arate că: S  ,
15
notațiile fiind cele cunoscute într-un triunghi. Emil C. Popa, Radu Diaconu, Sibiu
L:600. Să se rezolve ecuaţia: 2n{x}2  n 2 x  1 0, n  N .*
Gheorghe Ghiţă, Buzău
- PROBLEME PROPUSE -

L:601. Rezolvaţi în mulţimea numerelor reale nenule sistemul de ecuaţii:


­2 1
°x  2y 2( x 2  y 2 )
°
®
°2  1 y2  x2
°¯ x 2 y
D.M. Bătineţu-Giurgiu, Bucureşti şi Neculai Stanciu, Buzău
L:601. Fie (bn )n * un şir de numere reale în progresie geometrică de raţie q diferit de 1, atunci

demonstraţi egalitatea:

1 q q2 qi
n n
 n n
 n n
 ...  n n

¦ bk ˜ ¦ bk 1
k 1 k 1
¦ bk 1 ˜ ¦ bk 2
k 1 k 1
¦ bk 2 ˜ ¦ bk 3
k 1 k 1
¦ bk i ˜¦ bk i1
k 1 k 1

q i 1  1 1
˜ , n  N * , i  N . Constantin Ciobîcă , Fălticeni, Suceava
q 1 n n

¦b ˜ ¦b
k 1
k
k 1
k  i 1

L:602. Fie ABC un triunghi oarecare , wa lungimea bisectoarei interioare corespunzătoare laturii a=BC a
a b c a 2  b2  c 2
triunghiului dat , etc. Demonstrați că :   t2 3 . Vasile Jiglău , Arad
wa wb wc ab  bc  ca

L:603. Fie ABC un triunghi dreptunghic în A . Pe ipotenuza > BC @ considerăm punctul D astfel încât
CD 3
. Ştiind că m( CDA) 450 , să se demonstreze că AB 2 ˜ AC . Titu Zvonaru, Comăneşti
DB 2

ƒ Clasa a X-a
L:604. Folosind eventual identitatea x3  19 x  30 ( x  3)( x  2)( x  5) rezolvați ecuațiile
a) 3  19 ˜ 3  30
3x x
0; b) lg x  19lg x  30 0 .
3
Constantin Dinu, Buzău

§ x2 · x2 1
L:605. Să se arate că ecuația log16 ¨  2¸ nu are soluții reale. Adrian Stan, Buzău
© 2 ¹ 2 x2
L:606. Dacă a, b, z1 , z2 , z3  atunci,
z1  z2 ˜ az1  az2  b d z2  z3 ˜ az2  az3  b  z3  z1 ˜ az3  az1  b . Gheorghe Ghiţă, Buzău
­
° x  y 13
3 3

L:607. Rezolvaţi în mulţimea u sistemul de ecuaţii ® .


°̄ x 2 y  xy 2 4
D.M. Bătineţu-Giurgiu, Bucureşti şi Neculai Stanciu, Buzău
L:608. Rezolvați ecuația 29  31  1961 29x ˜ 31x  29x ˜1961x  31x ˜1961x .
2x 2x 2x

Mircea Mario Stoica, Arad


ax
L:609. Rezolvați ecuația log a2 ( x 2  2ax  2a 2 ) , a²1 . Marin Chirciu, Pitești
x  ax  a 2
2
- PROBLEME PROPUSE -

§ 1
· 7S
¸  artcg 3
x
x
L:610. Să se rezolve ecuația: arctg ¨ 32 . Ovidiu Țâțan, Râmnicu Sărat
© ¹ 12
(4n)!
L:611. Pentru n  * fixat , să se rezolve ecuația : Cxn ˜ Cxn n ˜ Cxn 2 n .
4 ˜ (n !)4
Ionel Tudor, Călugăreni, Giurgiu
( x  y ) x 2  xy  y 2
L:612. Fie x, y, z² 0 astfel încât ( x  y)( y  z )( z  x) 1 . Arătați că ¦ x  y  2 xy t 3 . În
ce caz avem egalitate ?
Marian Cucuoaneș, Mărășești, Lucian Tuțescu , Craiova
­ n  p 1 kp
° x ˜ C n p 2  k  p  1 ˜ y D ˜ n  p  1
k  p 1

°
L:613. Să se rezolve sistemul de ecuații: ® ,
° x ˜ C k  p2  n  p  1 ˜ y E ˜ k  p  1
n p 2
°¯ kp n  p 1
unde n, p, k  N , n t k  1, D , E  R.
Radu Diaconu, Sibiu
L:614. Dacă ABC este un triunghi cu notaţiile uzuale, atunci aflaţi mulţimea acestor triunghiuri care
verifică relaţia b  c t 2a .
2 2 2

Constantin Rusu, Râmnicu Sărat


L:615. Să se demonstreze că în orice triunghi ascuţitunghic ABC este adevărată inegalitatea:
9
¦ sin A sin B cos C d 8 . Neculai Stanciu, Buzău şi Titu Zvonaru, Comăneşti

L:616. Dacă a, b, c sunt numere reale strict pozitive, arătați că are loc inegalitatea:
ab bc ca a 2  b2  c 2 2
   d ˜ a  b  c . Florin Stănescu, Găești, Dâmbovița
a  b b  c c  a 2 ab  bc  ca 3

ƒ Clasa a XI-a
1 1 1
L:617. Se consideră determinantul: d a 1 3b 0; a, b  .
3b a 1
a) Să se calculeze d . b) Să se rezolve ecuația d 0.
c) Dacă a, 3b sunt rădăcinile ecuației x 2  2 x  2 0 , să se calculeze d .
Radu Diaconu , Sibiu
§ 50 53 ·
L:618. Fie matricea X ¨ ¸ . Determinați x, y  astfel încât X 2 x ˜ X  y ˜ I2 .
© 27 29 ¹
Doina și Mircea Mario Stoica, Arad
§1 1 2· § 2017 · § x·
¨ ¸ ¨ ¸ ¨ ¸
L:619 . Se consideră matricele A ¨ 0 1 3 ¸ , B ¨ 1 ¸ și X ¨ y ¸ cu x, y, z  .
¨0 0 0¸ ¨ 0 ¸ ¨z¸
© ¹ © ¹ © ¹

Să se rezolve ecuația A2018 ˜ X B. Constantin Dinu, Buzău


- PROBLEME PROPUSE -

§ a b 0·
L:620. Să se calculeze A n , unde A ¨ 0 a 0 ¸, a z e. Gheorghe Ghiţă, Buzău
¨ c d e¸
© ¹

L:621. Fie n² m²1 și matricele A, B  M 2 ( ) care îndeplinesc condițiile:


(i) det( A  n ˜ B) det(n ˜ A  B) ;
(ii) det( A  m ˜ B) det(m ˜ A  B) . Să se demonstreze că det( A) det( B) .
Marin Chirciu,Pitești
L:622. Fie a  2; 2 fixat, iar f : o , f ( x) 2  2x
. Să se arate că f ( )  >3;
x2 ( a  2) x 3
3 f .
Adrian Stan, Buzău
n
1
L:623. Fie s n 2 n  ¦ , lim s n s (constanta lui Ioachimescu).
n of
k k 1

Claculaţi lim ( s n  s) (2n  1)!! .


2 n
nof
D.M. Bătineţu-Giurgiu, Bucureşti şi Neculai Stanciu, Buzău
tg 2 a  1 §S S ·
L:624. Să se demonstreze inegalitățile : 2  ln(tg a) ¢ 2
¢ 2  tg 4 a ˜ ln(tg 2 a), a  ¨ , ¸ .
ln(tga) ©4 2¹

Ionel Tudor , Călugăreni , Giurgiu


L:625. Fie a  0;1 și un șir cu și xn a 2  a  xn1  2a a  xn1 , n t 1 .
Arătați că este convergent și calculați lim xn .
n of
Lorena – Luiza Cremeneanu , Constantina Prunaru , Craiova

ƒ CLASA a XII-a

L:626. Fie funcția f: o , f ( x) e x ˜ sin 2 x  2018  1 ,


 sin 2 x  e x
a) Arătați că f ( x)  f I ( x) 1  e x ˜ sin 2 x ; b) Calculați I ³ sin 2 x  e x  2018dx .
Constantin Dinu, Buzău

x2  x  1
L:627. Să se determine funcțiile derivabile F : 0, f o cu proprietatea F '
x
x 1
, x ! 0.
Ovidiu Tâțan, Rm. Sărat

1
e x (e 2  1)(9  2S 3 )
L:628. Să se arate că: ³0 x 2  x  1dx d
3 3e
.

Radu Diaconu , Sibiu

L:629. Determinați primitivele funcției f : o , f ( x) cos3 x ˜ cos ax, x  , unde a  , a este dat.
Marin Chirciu, Pitești
- PROBLEME PROPUSE -

L:630. Fie f :[a, b] o o funcţie pozitivă, derivabilă cu derivata continuă care are proprietatea
b
f (a)
f (b) . Să se demonstreze că ³ (2 x 3  3(a  b) x 2  6abx f c( x)dx t (a  b) 3 f (a) .
a
Constantin Rusu, Râmnicu Sărat

L:631. Dacă f : o este o funcţie de două ori derivabilă cu proprietatea că există a < b, astfel încât
f(a) = f(b) (f este neinjectivă), atunci există c‫(ג‬a; b) astfel încât:
c

³
f (c) f ( x)dx f ' (c)[ f (a)  3 f (c)].
''

a
Gheorghe Ghiţă, Buzău

(x  x2  4)n  (x  x2  4)n
2

³

L:632. Calculaţi: dx , (n  ).
0 x 4
2

D.M. Bătineţu-Giurgiu, Bucureşti şi Neculai Stanciu, Buzău

2S 1
L:633. Să se calculeze I ³0 sin x  cos4 x
4
dx . Gabriel Tănase, Buzău

L:634. Se consideră operaţia: X*Y = XY - YX, definită pe mulţimea matricelor pătratice de ordin n. Să se
arate că submulţimea M(A) = {XMn | AX = XA} pentru orice AM, A nesingulară, este parte stabilă în
raport cu operaţia indusă.

Petre Rău, Galați


L:635. Se consideră ecuația x  (2 x1  2 x2  1) x  ( x1  x2  2018) x  x1  x2  1 0 , care are toate
3 2

rădăcinile reale x1 , x2 , x3 . Să se arate că : (1  x1 )(1  x2 ) 1  x3 ; Rezolvați ecuația în .


Ionel Tudor , Călugăreni , Giurgiu

L:636. Să se calculeze limita şirului (a n ) n t0 având termenul general:


3S
2 cos(2n  1) x
an ³S x
dx . Vasile Mircea Popa, Sibiu
2
(în legătură cu problema C.O: 5074 din Gazeta Matematică nr. 11/2009).

L:637. Să se calculeze limita şirului u n n t1 având termenul general:


1 1 1
un arg th  arg th  ...  arg th ,
n 1 n2 2n
unde prin argth x am notat funcţia inversă a funcţiei tangentă hiperbolică (în legătură cu problema 26295 din
G.M. nr. 4/ 2010). Vasile Mircea Popa, Sibiu

De unde se nasc erorile? Este de ştiut că doar din faptul că voinţa fiind cu mult mai amplă şi
mai întinsă decât intelectul, ea nu se înglobează între aceleaşi limite, ci se extinde şi la lucruri pe care
nu le înţelege, care fiindu-i în sine indiferente, o fac să se rătăcească cu extremă uşurinţă, şi alege
răul în locul binelui sau falsul pentru adevăr.
Descartes
- QUICKIES-

„In order to improve the mind, we ought


less to learn, then to contemplate”.
Rene Descartes

5. QUICKIES
A Quickie should have an unexpected, succinct solution. Submitted quickies should not be under
consideration for publication elsewhere. We invite readers to submit solutions-quickies and new
proposals-quickies, accompanied by solutions mailed electronically (ideally MS Word 2003 or PDF
file) to stanciuneculai@yahoo.com. All communications should include the reader’s name, full address,
and an e-mail address. Submited solutions should arrive before October 31, 2018.

PROPOSALS – QUICKIES
Q41. Proposed by Mihàly Bencze, Bucharest, Romania.
§ S· 8 3
If x  ¨ 0,¸ , then prove that  t 14 .
© 2¹ sin 2 x sin x  cos 4 x
2 4

Q42. Proposed by Mihàly Bencze, Bucharest, Romania.


az  by x
If x, y, z ! 0 and x  y  z 1 , then prove that ¦ x  yz t ( a  b) ¦
x  yz
, for all a, b ! 0 .

Q43. Proposed by D.M. Bătineţu-Giurgiu, Bucharest, Romania.


If ABC is a triangle with area S and usual notations, then prove that
(a 4  1)(b 4  1)  (b 4  1)(c 4  1)  (c 4  1)(a 4  1) t 8 3S .
Q44. Proposed by D.M. Bătineţu-Giurgiu, Bucharest, Romania.
Let f : R o R be a continuos and odd function and g : R* o R be a continuos function such that
2 1
1
§1·
g¨ ¸
© x¹
 g ( x), x  R* . Compute ³ (1  x ) 1 2
a
( f D g )( x )

dx , where a ! 1 .
2 1

SOLUTIONS – QUICKIES
Q37. Proposed by Kevin Soto Palacios, Huarmey, Peru.
Let ABC be an acuted triangle. Prove that
9 3
tan A  tan B  tan C  sin 2 A  sin 2 B  sin 2C t .
2
Solution by Marin Chirciu, Piteşti, Romania. Using well-known identities
2rp 2rp
¦ tanA p  2R  r
2 2
and ¦ sin 2 A R2
,

2rp 2rp 9 3 1 1 9 3
the inequality becomes  t œ 2  2t œ
p  2R  r p  2R  r R
2 2 2 2
R 2 4rp
1 1 9˜ p 3
œ  t . Taking into account by
p2  2R  r
2
R2 4rp 2
4R  r t p 3 (Doucet inequality); p 2 t 2R2  8Rr  3r 2 (Walker inequality, in acute triangle );
- QUICKIES -

(Yang Xue Zhi inequality), it suffices to show that:


1 1 9 4R  r
 t œ
r R  2r 4r 2 R 2  8Rr  3r 2
2 2
R
 2R  r
2
4 R 2  4 Rr  3r 2 
Rr
R  Rr  r
2 2
9 4R  r
œ t œ 8R4 12R3r  21R2r 2  20Rr 3  12r 4 t 0 œ
2 2
Rr 4r 2 R  8Rr  3r
2 2

œ R  2r 8R3  4R 2r  13Rr 2  6r 3 t 0 , true by Euler inequality R t 2r .


The equality occurs iff triangle ABC is equilateral.
Solution by author. Lemma. If x, y, z ! 0 , such that xy  yz  zx 1 , then
1 4 9 3
 t .
xyz ( x  y )( y  z )( z  x) 2
AM GM 1 1
Indeed, 1 xy  yz  zx t 33 x 2 y 2 z 2 œ t x2 y2 z2 œ t 3 3 , (1).
27 xyz
AM GM
(2 xy  2 yz  2 zx) 3 ( xz  yz )  ( yx  zx)  ( zx  yx) t 27( xz  yz )( yx  zx)( zx  yx)
3
8
1 27 1 4
œ t , (2). 
( xz  yz )( yx  zx)( zx  yx ) 8 xyz ( x  y )( y  z )( z  x)
1 1 4 AM GM 1 1 4
  t 33 ˜ ˜
2 xyz 2 xyz ( x  y )( y  z )( z  x) 2 xyz 2 xyz ( x  y )( y  z )( z  x)

(1)
1 1 27 3 9 3
33 ˜ t 33 3 3 ˜ 3 3˜ .
xyz ( xz  yz )( yx  zx)( zy  xy ) ( 2 ) 8 2 2
§ S·
Now we make the trigonometric substitutions x cot A, y cot B, z cot C , A, B, C  ¨ 0, ¸ and
© 2¹
A B C S . By Lemma and the facts
1 sin A sin B 1 sin B sin C 1 sin C sin A
, , ,
cot A  cot B sin C cot B  cot C sin A cot C  cot A sin C
and the formulas
tan A  tan B  tan C tan A tan B tan C , respectively sin 2 A  sin 2B  sin 2C 4 sin A sin B sin C
1 4 9 3
we obtain  t œ
cot A cot B cot C (cot A  cot B)(cot B  cot C )(cot C  cot A) 2
9 3 9 3
œ tan A tan B tan C  4 sin A sin B sin C t œ tan A  tan B  tan C  sin 2 A  sin 2 B  sin 2C t
2 2
9 3
œ tan A  tan B  tan C  sin 2 A  sin 2 B  sin 2C t , q.e.d
2
Also solved by Titu Zvonaru, Comăneşti, Romania and Marius Drăgan, Bucharest, Romania.

Q38. Proposed by Mihàly Bencze, Bucharest, Romania.


Prove that if a, b, c ! 0 , then ¦ a §¨ ¦ a1 ·¸ t ¦ a §¨ ¦ 1a ·¸ .
8
8
© ¹ © ¹
Solution by Titu Zvonaru, Comăneşti, Romania.
- QUICKIES -

Using the inequality x 2  y 2  z 2 t xy  yz  zx , we have


a2 b2 c2 a b b c c a a b c
  t ˜  ˜  ˜   and
b2 c2 a2 b c c a a b c a b
a2 b2 c2 a b b c c a b c a
  t ˜  ˜  ˜   .
c2 a2 b2 c a a b b c c a b
Yields

¦ a §¨ ¦ a1 ·¸
2
2
3
a2 b2 c2 a2 b2 c2
    
b2 c2 a2 c2 a2 b2
t
© ¹
a b c b c a
t 3      ¦ a §¨ ¦ 1 ·¸ .
c a b c a b © a¹
Then, we obtain

¦ a §¨ ¦ a1 ·¸ t ¦ a §¨ ¦ a1 ·¸ t ¦ a §¨ ¦ a1 ·¸ t ¦ a §¨ ¦ a1 ·¸ , Q.E.D.
8
8
4
4
2
2
© ¹ © ¹ © ¹ © ¹

Remark. By induction yields ¦ a §¨ ¦ a1


2n
n
· § 1·
¸ t ¦ a ¨ ¦ ¸ , n  N .
© a¹
© 2
¹
Also solved by Marius Drăgan, Bucharest, Romania.

Q39. Proposed by D.M. Bătineţu-Giurgiu, Bucharest, Romania.


§ S· § § b ˜ n 1 (n  1)! · ·
If a  ¨ 0, ¸ and b arcsin a , then calculate lim n n!¨ sin¨ ¸  a¸.
© 2¹ n of ¨ ¨ n
n! ¸ ¸
© © ¹ ¹
n
Solution by Marius Drăgan, Bucharest, Romania. We have lim e.
n of n
n!
n 1 ( n  1)! § n 1 (n  1)! n n  1 · 1 e
If we denote u n ; lim u n lim ¨ ˜n ˜ ¸ ˜ ˜ 1 1 so
n n of n of ¨ n  1 n ¸ e 2
n! © n ! ¹
un  1 (n  1)! 1 n 1
lim 1 ; lim u nn lim ˜ lim e.
n of ln u
n
n of n of n! n 1 ( n  1 )! n o f n 1 ( n  1)!
§ § bn 1 (n  1)! · ·
Therefore x n n
n!¨ sin¨ ¸  a¸ n
n!(sin bu n  sin b)
¨ ¨ n
n! ¸ ¸
© © ¹ ¹
b
sin (u n  1)
b b 2 b
2 ˜ n n! ˜ sin (u n  1) ˜ cos (u n  1) b ˜ n n! ˜ ˜ cos (u n  1) ˜ (u n  1)
2 2 b 2
(u n  1)
2
b
sin (u n  1)
n
n! 2 b u 1
b˜ ˜ ˜ cos (u n  1) ˜ n ˜ ln u nn .
n b 2 ln u n
(u n  1)
2

n of
1
e n o f

Hence, lim xn b ˜ ˜ 1 ˜ cos b ˜ 1 ˜ ln lim u nn
b cos b
e
1 a2
a
arcsin a .

Q40. Proposed by D.M. Bătineţu – Giurgiu, Bucharest, Romania.


Consider a triangle ABC with sides a, b, c , medians ma , mb , mc , interior bisectors wa , wb , wc and the
area F .
- QUICKIES -

§ wa2 wb2 wc2 · 27 3


Prove that (ma2  mb2  mc2 )¨¨   ¸t F.
©a
2
b 2 c 2 ¸¹ 4
Solution by Titu Zvonaru, Comăneşti, Romania.
3 2 IonescuWeitzenbock 3
We have ma2  mb2  mc2 (a  b 2  c 2 ) t ˜ 4 3F 3 3F , (1).
4 4
w2 h2
Also we have wa t ha , wb t hb , wc t hc , so ¦ 2a t ¦ a2 , (2).
cyc a cyc a

h 1
By, a ha (ctgB  ctgC ) Ÿ a and analogously yields that
a ctgB  ctgC
2
§ ha · 1
¦ ¨ ¸
cyc © a ¹
¦ (ctgA  ctgB )
cyc
2
, (3).

9 1
By Iranian’s inequality ( xy  yz  zx)
cyc
¦ ( x  y)
4
t
, x, y, z ! 0 , where we put
2

1 9
x ctgA, y ctgB , z ctgC and using ¦ ctgActgB 1 we deduce ¦ t , (4).
cyc (ctgA  ctgB )
2
cyc 4
§ wa2 wb2 wc2 · 9 27 3
By (1), (2), (3) and (4) we obtain (m  m  m )¨¨ 2  2  2 ¸¸ t ˜ 3 3F
2
a
2
b
2
c F , q.e.d.
©a b c ¹ 4 4
Equality occurs if triangle ABC is equilateral

Also solved by Marius Drăgan, Bucharest, Romania.

CALEIDOSCOP MATEMATIC

P1. În figura alăturată centrul pătratului EFGH, cu lungimea


laturii de 4 cm, este situat în vârful A al pătratului ABCD. Fără a
ști care este lungimea laturii pătratului ABCD sau măcar a vreunui
unghi făcut de o latură a pătratului ABCD cu o latură a pătratului
EFGH, putem oare noi calcula aria suprafeței hașurate ?

P2. În figura alăturată sunt patru triunghiuri dreptunghice congruente


având dimensiunile ca în figură. Cu cât este egală aria unui triunghi?

Răspunsuri la pag 59.


- CALEIDOSCOP MATEMATIC -

„ Aceia care merg foarte încet, dacă ţin drumul drept,


pot înainta mult mai departe decât cei care merg repede,
dar se abat de la acest drum. ”.
Descartes
(1596- 1650)

7. Caleidoscop matematic
1. Inegalităţi aproape fără cuvinte
D.M. Bătineţu-Giurgiu, Bucureşti, Dan Sitaru, Drobeta Turnu-Severin, Neculai Stanciu, Buzău

Inegalitatea Tsintsifas: Dacă x, y, z ! 0 , atunci în orice triunghi ABC :


x y 2 z
a2  b  c 2 t 2 3 ˜ [ ABC ] , (T).
yz zx x y
Egalitatea are loc dacă şi numai dacă x y z şi a b c .
x y z 3
Inegalitatea Nesbitt-Ionescu:Dacă x, y, z ! 0 , atunci:   t , (N-I).
yz zx x y 2
Egalitatea are loc dacă şi numai dacă x y z .
Inegalitatea Ionescu-Weitzenböck: Dacă ABC este un triunghi, atunci:
a 2  b 2  c 2 t 4 3 ˜ [ ABC ] , (I-W). Egalitatea are loc dacă şi numai dacă a b c .
(T) Ÿ (N-I): În (T) luăm a b c .
(T) Ÿ (I-W): În (T) luăm x y z .

2. Numere naturale care se regăsesc în puterile lor


Prof. Kovacs Bela, Satu Mare
Cuburi perfecte, care se termină cu aceleași cifre, respectiv grupe de cifre identice cu cifrele bazei.
1. Numere cu o singură cifră: 03 0; 13 1; 43 64; 53 125; 63 216; 93 729.
2. Numare naturale cu două cifre:
243 13824; 253 15625; 493 117649; 513 132651; 753 421875; 763 437876; 993 970299 .
3. Numere naturale formate din trei cifre:
1253 1953125; 2493 15438249; 2513 15813251; 3753 52734375; 3763 53157376 ,
4993 124251499; 6243 242970624; 6253 244140625; 7493 420189749; 7513 423564751 ,
8753 669921875; 9993 997002999; .
4. Numere naturale cu patru cifre:
12493 = 1.948.441.249 37513 = 52.776.573.751 43753 = 83.740.234.275
49993 = 124.925.014.999 56253 = 177.978.515.625 87513 = 670.151.588.751
93753 = 823.974.609.375 93763 = 824.238.309.376 99993 = 999.700.029.999

5. Numere naturale cu cinci cifre:


18.7513 = 6.592.851.618.751 31.2493 = 30.514.648.531.249
40.6253 = 67.047.119.140.625 49.9993 = 124.992.500.149.999
59.3753 = 209.320.068.359.375 68.7513 = 324.965.351.768.751
81.2493 = 536.357.148.681.249 90.6253 = 744.293.212.890.625
99.9993 = 999.970.000.299.999
- CALEIDOSCOP MATEMATIC -

6. Numere naturale cu șase cifre:


109.3753 = 1.308.441.162.109.375 109.3763 = 1.308.477.051.109.376
281.2493 = 22.247.077.149.281.249 390.6253 = 59.604.644.775.390.625
499.9993 = 124.999.250.001.499.999 609.3753 = 226.284.027.099.609.375
781.2493 = 476.835.327.150.781.249 890.6253 = 706.455.230.712.890.625

Generalizare: Tot așa mai departe, putem obține astfel de numere, din ce în ce mai mari. De
exemplu, un număr de 10 cifre, cu astfel de proprietate este:
1.787.109.3763 = 5.707.598.300.918.769.841.787.109.376
Presupunere: Există numere oricât de mari, cu proprietatea, că puterea a treia se termină cu
aceași grupă de cifre ca și cifrele bazei.

3. Un împărat a dorit să-l pună la încercare pe înțeleptul curții supunându-l la o probă de


încercare. Astfel, împăratul a cerut să i se aducă două urne în care, în una erau cincizeci de
bile albe iar în cealaltă erau cinczeci de bile negre. Înțeleptului curții i s-a cerut să aranjeze
cum vrea el bilele în cele două urne astfel, atunci când va extrage o bilă fără să vadă bilele și
cele două urne de unde extrage, dacă va ieși o bilă albă va primi un dar prețios dar dacă va
scoate o bilă neagră va trebui să părăsească curtea.
Cum a procedat înțeleptul ca să-și maximizeze șansa de a extrage o bilă albă și astfel de a rămâne
la curte?
Rezolvare:
Înțeleptul curții a procedat astfel: În prima urnă a lăsat o singură bilă albă iar în a doua urnă, pe lângă
cele 50 de bile negre le-a adăugat și pe celelalte 49 de bile albe.
Astfel, în prima urnă există o bil albă iar în a doua urnă sunt 99 bile din care 49 albe iar 50 negre.
Problema revine la calculul unei probabilități totale adică, dacă considerăm următoarele evenimente:
A- reprezintă evenimentul ca bila extrasă să fie albă;
A1- reprezintă evenimentul ca extragerea să fie făcută din prima urnă;
A2- reprezintă evenimentul ca extragerea să fie făcută din a doua urnă.
Atunci, probabilitatea ca bila extrasă să fie albă este dată de
1 1 49
P( A) P( A1 ) ˜ PA1 ( A)  P( A2 ) ˜ PA2 ( A) ˜1  ˜ 0,5  0,5 ˜ 0, 49 0,5  0, 245 0,745 .
2 2 99

Răspunsurile problemelor de la pag. 57.


P1.
Se prelungesc dreptele BA și DA până intersectează FE respectiv FG.
Astfel că pătratul EFGH este format din patru patrulatere congruente care au
aceeași arie datorită proprietății de simetrie a pătratului față de centrul său.
Aria patrulaterul hașurat este o pătrime din aria pătratului EFGH, adică 16:4
= 4 cm2.

P2. b) Cele patru triunghiuri se translatează ca în figura alăturată


obținându-se un pătrat de latură 3 iar în centru un pătrat de latură 1. Atunci,
32 12  4 A' Ÿ A' 2.
- POȘTA REDACȚIEI-

„ Adevăratul defect e să ai
defecte și să nu le îndrepțí”.
(Confucius
(551- 479)

8. Poşta redacţiei
Dragi cititori, elevi şi profesori, a apărut numărul 21 al revistei de matematică „ SCLIPIREA
MINTII”, o revistă care promovează studiul matematicii în rândul elevilor noştri, şi care, sperăm noi, va
aduna tot mai mulţi elevi şi profesori împreună, pentru a face din obiectul matematicii o activitate atractivă și
performantă.

Profesorii şi elevii care doresc să trimită materiale pentru revistă, constând în articole, exerciţii şi
probleme cu enunţ şi rezolvare completă, materiale pentru „caleidoscop matematic”, sau orice alte sugestii
pentru a îmbunătăţii calitatea acestei reviste, o pot face trimiţând materialele membrilor colectivului de
redacţie sau pe adresa de e_mail: ady_stan2005@yahoo.com, fie materiale tehnoredactate( salvate în
Word 2003) , fie scrise de mână şi scanate. Materialele primite trebuie să fie originale şi să nu mai fi fost
trimise sau să mai fie trimise şi către alte reviste. Dreptul de autor al materialelor trimise spre publicare,
aparţine redacţiei.

Data finală până când profesorii pot trimite materialele, rezolvările şi comenzile pentru numărul 22 al
revistei „ SCLIPIREA MINTII” va fi 15 Octombrie 2018. Vă urăm succes şi vă aşteptăm.

ELEVI REZOLVITORI
Școala Gimnazială “Gh. Popescu” Mărgineni- Slobozia, Scornicești, Olt
Clasa a VI-a: 28p. Mindreanu Florentina, Vlaicu Liliana; Clasa a VII-a: 35 p. Sima Iuliana, 27p. Ionescu
Miruna, 25p. Screciu Constantin; Prof. Iuliana Trașcă.

Școala Gimnazială Padina, Buzău


Clasa a V-a: 19p. Andronache Oana, Spătaru Alexandra, 18p: Iacob Mariana, 16p. Filipoiu Denisa, Spătaru
Denisa.
Clasa a VI-a: 29p. Hermeneanu Lavinia, 27. Balica Costin, 26p. Spătaru Maria; Iasăgeanu Gabriela,
Clasa a VII-a: 29p. Dinu Cristina, 28p. Dumitroiu Elena, 26p. Radu Mihaela, Jega Mihaela, 8p. Clasa a
VIII-a: 29p. Nedelcu Daniela, 28p. Ceauș Andreea, 26p. Fogor Antonia, 25p. Burlacu Alin, 8p. Păltinațu
Laura; Prof. Stănescu Ion.
Școala Gimnazială ” Rareș Vodă” Ploiești, Prahova
Clasa a V-a: 20p. Păduraru Cătălina, Mihai Alexandra, Găitănaru Andreea, Călinescu Alin, 19p Niţoiu Vlad,
8p Boboc Gabriel, Savu Gabriel
Clasa a VIII-a: 25p. Radu Elena, Săvulescu Ionuţ, 23p. Stănescu Alina, Nicolae Andreea. Prof. Daniela
Badea

S-ar putea să vă placă și